You are on page 1of 84

UNIVERSITY OF MINDANAO

College of Engineering Education


Computer Engineering Program

Physically Distanced but Academically Engaged

Self-Instructional Manual (SIM) for Self-Directed Learning (SDL)

Course/Subject:
CEE109 - Engineering Economy
Author:
Engr. Marianne G. Wata

THIS SIM/SDL MANUAL IS A DRAFT VERSION ONLY; NOT FOR REPRODUCTION AND
DISTRIBUTION OUTSIDE OF ITS INTENDED USE. THIS IS INTENDED ONLY FOR THE USE
OF THE STUDENTS WHO ARE OFFICIALLY ENROLLED IN THE COURSE/SUBJECT.
EXPECT REVISIONS OF THE MANUAL.
College of Engineering Education
2nd Floor, B&E Building
Matina Campus, Davao City
Telefax: (082) 296-1084
Phone No.: (082)300-5456/300-0647 Local 133

TABLE OF CONTENTS

PAGE
Cover Page ………………………………………………………………………………………………. 1
Table of Contents………………………………………………………………………………………. 2
Course Outline…………………………………………………………………………………………... 4
Course Outline Policy………………………………………………………………………………… 4
Course Information…………………………………………………………………………………… 7

Topic/ Activity
Unit Learning Outcomes- Unit 1…………………………………………………………………. 8
Big Picture in Focus: ULO-1a…………………………………………………………………..….. 8
Metalanguage…………………………………………………………………………………... 8
Essential Knowledge………………………………………………………………………… 9
Self-Help………………………………………………………………………………………….. 16
Let’s Check……...……………………………………………………………………………….. 17
Let’s Analyze...………………………………………………………………………………….. 18
In a Nutshell…………………………………………………………………………………….. 20
Q & A List ……………………………………………………………………………………...… 21
Keyword Index.……………………………………………………………………………….. 21
Big Picture in Focus: ULO-1b…………………………………………………………………..….. 23
Metalanguage………………………………………………………………………………….. 23
Essential Knowledge………………………………………………………………………… 23
Self-Help………………………………………………………………………………………….. 36
Let’s Check……...……………………………………………………………………………….. 37
Let’s Analyze...………………………………………………………………………………….. 37
In a Nutshell…………………………………………………………………………………….. 38
Q & A List ………………………………………………………………………………………... 39
Keyword Index.………………………………………………………………………………… 39

ii
College of Engineering Education
2nd Floor, B&E Building
Matina Campus, Davao City
Telefax: (082) 296-1084
Phone No.: (082)300-5456/300-0647 Local 133

Unit Learning Outcomes- Unit 2…………………………………………………………………. 40


Big Picture in Focus: ULO-2a…………………………………………………………………..….. 40
Metalanguage…………………………………………………………………………………... 40
Essential Knowledge………………………………………………………………………… 40
Self-Help………………………………………………………………………………………….. 47
Let’s Check……...……………………………………………………………………………….. 47
Let’s Analyze...………………………………………………………………………………….. 48
In a Nutshell…………………………………………………………………………………….. 48
Q & A List ……………………………………………………………………………………...… 49
Keyword Index.……………………………………………………………………………….. 49
Big Picture in Focus: ULO-2b…………………………………………………………………..….. 50
Metalanguage………………………………………………………………………………….. 50
Essential Knowledge………………………………………………………………………… 50
Self-Help………………………………………………………………………………………….. 53
Let’s Check……...……………………………………………………………………………….. 53
Let’s Analyze...………………………………………………………………………………….. 54
In a Nutshell…………………………………………………………………………………….. 54
Q & A List ………………………………………………………………………………………... 55
Keyword Index.………………………………………………………………………………… 55
Unit Learning Outcomes- Unit 3…………………………………………………………………. 56
Big Picture in Focus: ULO-3a…………………………………………………………………..….. 56
Metalanguage…………………………………………………………………………………... 56
Essential Knowledge………………………………………………………………………… 57
Self-Help………………………………………………………………………………………….. 65
Let’s Check……...……………………………………………………………………………….. 65
Let’s Analyze...………………………………………………………………………………….. 66
In a Nutshell…………………………………………………………………………………….. 67
Q & A List ……………………………………………………………………………………...… 68
Keyword Index.……………………………………………………………………………….. 68

iii
College of Engineering Education
2nd Floor, B&E Building
Matina Campus, Davao City
Telefax: (082) 296-1084
Phone No.: (082)300-5456/300-0647 Local 133

Big Picture in Focus: ULO-3b…………………………………………………………………..….. 69


Metalanguage………………………………………………………………………………….. 69
Essential Knowledge………………………………………………………………………… 69
Self-Help………………………………………………………………………………………….. 71
Let’s Check……...……………………………………………………………………………….. 72
Let’s Analyze...………………………………………………………………………………….. 72
In a Nutshell…………………………………………………………………………………….. 73
Q & A List ………………………………………………………………………………………... 74
Keyword Index.………………………………………………………………………………… 74
Unit Learning Outcomes- Unit 4…………………………………………………………………. 75
Big Picture in Focus: ULO-4a…………………………………………………………………..….. 75
Metalanguage…………………………………………………………………………………... 75
Essential Knowledge………………………………………………………………………… 75
Self-Help………………………………………………………………………………………….. 77
Let’s Check……...……………………………………………………………………………….. 77
Let’s Analyze...………………………………………………………………………………….. 78
In a Nutshell…………………………………………………………………………………….. 78
Q & A List ……………………………………………………………………………………...… 79
Keyword Index.……………………………………………………………………………….. 79
Big Picture in Focus: ULO-4b…………………………………………………………………..….. 80
Metalanguage………………………………………………………………………………….. 80
Essential Knowledge………………………………………………………………………… 80
Self-Help………………………………………………………………………………………….. 82
Let’s Check……...……………………………………………………………………………….. 82
Let’s Analyze...………………………………………………………………………………….. 83
In a Nutshell…………………………………………………………………………………….. 83
Q & A List ………………………………………………………………………………………... 84
Keyword Index.………………………………………………………………………………… 84

iv
College of Engineering Education
2nd Floor, B&E Building
Matina Campus, Davao City
Telefax: (082) 296-1084
Phone No.: (082)300-5456/300-0647 Local 133

v
College of Engineering Education
2nd Floor, B&E Building
Matina Campus, Davao City
Telefax: (082) 296-1084
Phone No.: (082)300-5456/300-0647 Local 133

Course Outline: CEE 109 – Engineering Economy

Course Coordinator: Marianne G. Wata


Email: mwata@umindanao.edu.ph
Student Consultation: Online (LMS) or e-mail
Mobile: 0910 148 2260
Phone: (083) 553 8020
Effectivity Date: May 2020
Mode of Delivery: Online Blended Delivery
Time Frame: 54 hours
Student Workload: Expected Self-Directed Learning
Pre-requisite: 3rd Year Standing
Credit: 3.0 units
Attendance Requirements: For online (virtual/face-to-face) sessions: a
minimum of 95% attendance; for 1-day on-
campus/onsite review: 100% attendance; for 1-
day on-campus/onsite final exam: 100%
attendance

Course Outline Policy

Areas of Concern Details


Contact and Non-contact Hours This 3-unit course self-instructional manual is designed for
blended learning mode of instructional delivery with
scheduled face to face or virtual sessions. The expected
number of hours will be 54 including the face-to-face or
virtual sessions. The face-to-face sessions shall include
the summative assessment tasks (exams) since this
course is crucial in the licensure examination for
engineering.

Assessment Task Submission Submission of assessment tasks shall be on the 3rd, 5th,
7th and 9th week of the term. The assessment paper shall
be attached with a cover page indicating the title of the
assessment task, the name of the course coordinator, date
of submission and name of the student. The document
should be emailed to the course coordinator. It is also
expected that you already paid your tuition and other fees
before the submission of the assessment task.

If the assessment task is done in real time through the


features in the Blackboard Learning Management System,
the schedule shall be arranged ahead of time by the course
coordinator. If permitted by our current situation, final exam
must be held on-campus.

6
College of Engineering Education
2nd Floor, B&E Building
Matina Campus, Davao City
Telefax: (082) 296-1084
Phone No.: (082)300-5456/300-0647 Local 133

Penalties for Late The score for an assessment item submitted after the
Assignments/Assessments designated time on the due date, without an approved
extension of time, will be reduced by 5% of the possible
maximum score for that assessment item for each day or
part day that the assessment item is late.

However, if the late submission of assessment paper has


a valid reason, a letter of explanation should be submitted
and approved by the course coordinator. If necessary, you
will also be required to present/attach evidences.
Return of Assignments/ Assessment tasks will be returned to you two (2) weeks
Assessments after the submission. This will be returned by email or via
Blackboard portal.

For group assessment tasks, the course coordinator will


require some or few of the students for online or virtual
sessions to ask clarificatory questions to validate the
originality of the assessment task submitted and to ensure
that all the group members are involved.
Assignment Resubmission You should request in writing addressed to the course
coordinator your intention to resubmit an assessment task.
The resubmission is premised on the student’s failure to
comply with the similarity index and other reasonable
grounds such as academic literacy standards or other
reasonable circumstances e.g. illness, accidents, financial
constraints.
Re-marking of Assessment You should request in writing addressed to the program
Papers and Appeal coordinator your intention to appeal or contest the score
given to an assessment task. The letter should explicitly
explain the reasons/points to contest the grade. The
program coordinator shall communicate with the students
on the approval and disapproval of the request.

If disapproved by the course coordinator, you can elevate


your case to the program head or the dean with the original
letter of request. The final decision will come from the dean
of the college.
Grading System All culled from BlackBoard sessions and traditional
contact
Course discussions/exercises – 30%
1st formative assessment – 10%
2nd formative assessment – 10%
3rd formative assessment – 10%

All culled from on-campus/onsite sessions (TBA):


Final exam – 40%

Submission of the final grades shall follow the usual


University system and procedures.
Referencing Style APA

7
College of Engineering Education
2nd Floor, B&E Building
Matina Campus, Davao City
Telefax: (082) 296-1084
Phone No.: (082)300-5456/300-0647 Local 133

Student Communication You are required to have a umindanao email account


which is needed to access the BlackBoard portal. Then,
the course coordinator shall enroll, you, the students to
have access to the materials and resources of the course.
All communication formats: chat, submission of
assessment tasks, requests etc. shall be through the portal
and other university recognized platforms.
You can also meet the course coordinator in person
through the scheduled face-to-face sessions to raise your
issues and concerns.
For students who have not created their student email
account, please contact the course coordinator or program
head.
Contact Details of the Dean Dr. Charlito L. Cañesares
Email: clcanesares@umindanao.edu.ph
Phone: (082) 296-1084 or 300-5456 loc. 133
Contact Details of the Program Engr. Randy E. Angelia, MEP-ECE, MSCpE
Head Email: randy_angelia@umindanao.edu.ph
Phone: (082) 296-1084 local 133
Students with Special Needs Students with special needs shall communicate with the
course coordinator about the nature of his or her special
needs. Depending on the nature of the need, the course
coordinator with the approval of the program coordinator
may provide alternative assessment tasks or extension of
the deadline of submission of assessment tasks. However,
the alternative assessment tasks should still be in the
service of achieving the desired course learning outcomes.
Help Desk Contact CEE Blackboard Administrator:
Engr. Jetron A. Adtoon, MSCpE
Email: jadtoon@umindanao.edu.ph
Phone: +63 9055 267834

CEE:
Frida Santa O. Dagatan
Email: cee@umindanao.edu.ph
Phone: +63 9055 267834

GSTC:
Ronadora E. Deala, RPsy, RPm, RGC, LPT
Email: ronadora_deala@umindanao.edu.ph
Phone: +63 921 2122846

Silvino P. Josol
Email: gstcmain@umindanao.edu.ph
Phone: +63 906 0757721
Library Contact Brigida E. Bacani
Email: library@umindanao.edu.ph
Phone: +63 951 3766681

8
College of Engineering Education
2nd Floor, B&E Building
Matina Campus, Davao City
Telefax: (082) 296-1084
Phone No.: (082)300-5456/300-0647 Local 133

Course Information- see/download course syllabus in the BlackBoard LMS

CC’s Voice: Hello future engineers! Welcome to the course CEE109: Engineering
Economy. By this time, I am positive that you are already well-equipped with
basic and complex knowledge of mathematics, analytical skills, and English
comprehension since this course requires deep understanding in solving
intricate problems involving the study of economy. Also, I would be expecting
a dynamic and an interactive online session with you throughout the term
and I hope you’ll have a great time learning Engineering Economy.

CO: Before becoming a certified engineer, you must deal with Engineering
Economy first given that this will be included in your licensure examination soon.
To verify that you really have gained some important insights from this course,
you are expected to identify the concepts, principles, and laws in
engineering economy as part of a decision-making process, solve
economy equivalence using interest rates, formulas and factors,
depreciation and cash flows, and, lastly, perform constrained alternative
selection using different economy study methods and mutually exclusive
alternatives at the end of the term. Thus, you are encouraged to study in
advance the concepts of Engineering Economy using various materials and
resources that can be accessed through our e-library (refer to the Library Contact
Details for more information).

Let us begin!

9
College of Engineering Education
2nd Floor, B&E Building
Matina Campus, Davao City
Telefax: (082) 296-1084
Phone No.: (082)300-5456/300-0647 Local 133

Big Picture

Week 1-3: Unit Learning Outcomes-Unit 1 (ULO-1): At the end of the unit, you are
expected to:

a. Be able to introduce the basic principles in economy, supply and demand relationship;
b. Be able to determine the simple interest and compound interest, cash flow diagrams
and their applications to real life situations; and

Big Picture in Focus:


ULO-1a. Be able to introduce the basic principles in economy, supply and
demand relationship

Metalanguage
This section will serve as your word bank where the most essential terms relevant
to the introduction of engineering economy and ULO-1a will be operationally defined to
establish a common frame of reference. You will encounter these terms as we delve
deeper to the study of Engineering Economy. Please refer to these definitions in case
you will find it difficult to understand.

1. Engineering Economy. It is a discipline that is concerned with the economic aspect


of engineering. It includes the systematic evaluation of the economic advantages of
proposed solutions to the engineering problems. To be economically acceptable,
solutions to the engineering problem must demonstrate a positive balance of long-
term benefits over long-term cost.
2. Engineering Economics. It is the application of economic techniques to the
evaluation of design and engineering alternatives. The role of engineering
economics is to assess the appropriateness of a given project, estimate its value,
and justify it from an engineering perspective.
3. Engineering Economic Analysis. It is a study of the desirability of making an
investment. It is primarily inspired by the work that engineers do in performing
analyses, synthesizing, and concluding as they work on projects of all sizes.
Moreover, engineering economy is at the center of making decisions. These
decisions involve the fundamental elements of cash flows of money, time, and
interest rates.
4. Cost. The accounting profession has developed special terms for certain group of
costs:
4.1 When annual costs are incurred due to the functioning of a piece of
equipment, they are known as Operating and Maintenance (O&M) Costs.

10
College of Engineering Education
2nd Floor, B&E Building
Matina Campus, Davao City
Telefax: (082) 296-1084
Phone No.: (082)300-5456/300-0647 Local 133

4.2 The annual costs associated with operating a business (other than the costs
directly attributable to production) are known as General, Selling, and
Administrative (GS & A) Expenses.
4.3 Direct Labor Costs are costs incurred in the factory, such as assembly,
machining, and painting labor costs.
4.4 Direct Material Costs are the costs of all materials that go into production.
Typically, both direct labor and direct material costs are given on a per-unit
or per-item basis.
4.5 The sum of the direct labor and direct material costs is known as the Prime
Costs.
5. Expenses. In operating a business, these are the terms used for describing its
expenses:
5.1 Indirect Manufacturing Expenses (IME) Or Indirect Material and Labor
Costs are the costs of factory supervision, stock- picking, quality control, factory
utilities, and miscellaneous supplies (cleaning fluids, assembly lubricants,
routing tags, etc.) that are not incorporated into the final product.
5.2 Factory Cost is the sum of the per-unit indirect manufacturing expenses
and prime cost.
5.3 Research and Development (R&D) Costs and Administrative Expenses
are added to the factory costs to give manufacturing cost of the product.
5.4 Selling Expenses or Marketing Expenses is an additional cost are
incurred in marketing the product.
5.5 Total Cost is the sum of the selling expenses and manufacturing.

Essential Knowledge
To perform the aforesaid big picture (unit learning outcomes) for the first three
(3) weeks of this course, you need to fully understand the following essential knowledge
that will be laid down in the succeeding pages. Please note that you are not limited to
exclusively refer to these resources. Thus, you are expected to utilize other books,
research articles and other resources that are available in the university’s library.

1. Non-Quantifiable Factors. An engineering economic analysis is a quantitative


analysis. Some factors cannot be introduced as numbers into the calculations. Such
factors are known as non-quantitative factors, judgment factors, and irreducible
factors. Typical non-quantifiable factors are:

1. Preferences
2. Political ramifications
3. Urgency
4. Goodwill
11
College of Engineering Education
2nd Floor, B&E Building
Matina Campus, Davao City
Telefax: (082) 296-1084
Phone No.: (082)300-5456/300-0647 Local 133

5. Prestige
6. Utility
7. Corporate Strategy
8. Environmental Effects
9. Health and Safety Rules
10. Reliability
11. Political Risks

Since these factors are not included in the calculations, the policy is to disregard
the issues entirely. Of course, the factors should be discussed in a final report. The
factors are particularly useful in breaking ties between competing alternatives that are
economically equivalent.

2. Economics. It is concerned with the well-being of all people, including those with
jobs and those without jobs, as well as those with high incomes and those with low
incomes. Economics acknowledges that production of useful goods and services can
create problems of environmental pollution.
It explores the question of how investing in education helps to develop workers’
skills. It probes questions like how to tell when big businesses or big labor unions are
operating in a way that benefits society as a whole and when they are operating in a
way that benefits their owners or members at the expense of others.

3. Microeconomics and Macroeconomics. Economics is divided into two branches:


Microeconomics focuses on the actions of individual agents within the economy,
like households, workers, and businesses;
Macroeconomics looks at the economy as a whole. It focuses on broad issues
such as growth of production, the number of unemployed people, the inflationary
increase in prices, government deficits, and levels of exports and imports.
Microeconomics and macroeconomics are not separate subjects, but rather
complementary perspectives on the overall subject of the economy.

4. Demand for Goods and Services. It is a term used by economists to refer to the
amount of some goods or service consumers are willing and able to purchase at each
price. Demand is based on needs and wants—a consumer may be able to
differentiate between a need and a want, but from an economist’s perspective they
are the same thing.
Demand is also based on ability to pay. If you cannot pay for it, you have no
effective demand. What a buyer pays for a unit of the specific good or service is called
price. The total number of units purchased at that price is called the quantity
demanded.

12
College of Engineering Education
2nd Floor, B&E Building
Matina Campus, Davao City
Telefax: (082) 296-1084
Phone No.: (082)300-5456/300-0647 Local 133

A rise in price of a good or service almost always decreases the quantity


demanded of that good or service. Conversely, a fall in price will increase the quantity
demanded. This relationship is illustrated in Fig.1. When the price of a gallon of
gasoline goes up, for example, people look for ways to reduce their consumption by
combining several errands, commuting by carpool or mass transit, or taking weekend
or vacation trips closer to home.

Fig. 1a-1 General Price-Demand Relationship

5. Necessities and Luxuries. These are the main reasons why people find ways to
look for means in earning money.

Necessities are those products or services that are required to support human
life and activities that will be purchased in somewhat the same quantity even though
the price varies considerably.
Luxuries are those products or services that are desired by humans which will
be purchased if money is available after the necessities are obtained.

Fig. 1a-2 Price-Demand Relationship for Necessities and Luxuries

13
College of Engineering Education
2nd Floor, B&E Building
Matina Campus, Davao City
Telefax: (082) 296-1084
Phone No.: (082)300-5456/300-0647 Local 133

6. Supply for Goods and Services. Price is what the producer receives for selling one
unit of a good or service. A rise in price almost always leads to an increase in the
quantity supplied of that good or service, while a fall in price will decrease the quantity
supplied.

When the price of gasoline rises, for instance, it encourages profit-seeking firms
to take several actions: expand exploration for oil reserves; drill for more oil; invest in
more pipelines and oil tankers to bring the oil to plants where it can be refined into
gasoline; build new oil refineries; purchase additional pipelines and trucks to ship the
gasoline to gas stations; and open more gas stations or keep existing gas stations
open longer hours.

Fig. 1a-3 Price-Supply Relationship for Necessities and Luxuries

7. Ceteris Paribus Assumption. A demand curve or a supply curve is a relationship


between two, and only two, variables: quantity on the horizontal axis and price on the
vertical axis. The assumption behind a demand curve or a supply curve is that no
relevant economic factors, other than the product’s price, are changing. Economists
call this assumption ceteris paribus, a Latin phrase meaning “other things being
equal.” Any given demand or supply curve is based on the ceteris paribus assumption
that all else is held equal.

Fig. 1a-4 Assumption of Ceteris Paribus


14
College of Engineering Education
2nd Floor, B&E Building
Matina Campus, Davao City
Telefax: (082) 296-1084
Phone No.: (082)300-5456/300-0647 Local 133

8. The Law of Supply and Demand. The law of supply and demand may be stated as
follows:

“Under conditions of perfect competition, the price at which a given product will be
supplied and purchased is the price that will result in the supply and the demand
being equal”

Fig. 1a-5 Price-Supply-Demand Relationship

9. Price Ceilings and Price Floors. Laws that government enacts to regulate prices
are called price controls. Price controls come in two flavors, the price ceiling and the
price floor. A price ceiling keeps a price from rising above a certain level (the “ceiling”),
while a price floor keeps a price from falling below a certain level (the “floor”).
Price ceilings, in Fig. 2, are enacted to keep prices low for those who demand the
product. But when the market price is not allowed to rise to the equilibrium level,
quantity demanded exceeds quantity supplied, and thus a shortage occurs. Those
who manage to purchase the product at the lower price given by the price ceiling will
benefit, but sellers of the product will suffer, along with those who are notable to
purchase the product at all. Quality is also likely to deteriorate.

Fig. 1a-6 Price Ceiling

15
College of Engineering Education
2nd Floor, B&E Building
Matina Campus, Davao City
Telefax: (082) 296-1084
Phone No.: (082)300-5456/300-0647 Local 133

A price floor is the lowest legal price that can be paid in markets for goods and
services, labor, or financial capital. Perhaps the best-known example of a price floor is
the minimum wage, which is based on the normative view that someone working full
time ought to be able to afford a basic standard of living.

Fig. 1a-7 Price Floor

Price ceilings prevent a price from rising above a certain level. When a price
ceiling is set below the equilibrium price, quantity demanded will exceed quantity
supplied, and excess demand or shortages will result. Price floors prevent a price
from falling below a certain level. When a price floor is set above the equilibrium price,
quantity supplied will exceed quantity demanded, and excess supply or surpluses will
result. Price floors and price ceilings often lead to unintended consequences.
10. Consumer, Producer, and Social Surplus. Consumer surplus is the gap between
the price that consumers are willing to pay, based on their preferences, and the
market equilibrium price.
Producer surplus is the gap between the price for which producers are willing to
sell a product, based on their costs, and the market equilibrium price.
Social surplus is the sum of consumer surplus and producer surplus. Total
surplus is larger at the equilibrium quantity and price than it will be at any other
quantity and price. Deadweight loss is loss in total surplus that occurs when the
economy produces at an inefficient quantity.

Fig. 1a-8 Surplus

16
College of Engineering Education
2nd Floor, B&E Building
Matina Campus, Davao City
Telefax: (082) 296-1084
Phone No.: (082)300-5456/300-0647 Local 133

11. Fixed and Variable Costs. If a cost is a function of the independent variable, the
cost is said to be a Variable Cost. The change in cost per unit variable change (i.e,
usually called the slope) is known as the Incremental Cost. Material and labor costs
are examples of variable costs.

If a cost is not a function of the independent variable, the cost is said to be a Fixed
Costs. Rent and lease payments are typical fixed costs. A table showing other
examples of fixed and variables costs is displayed on Table 1a-1.

Table 1a-1. Fixed and Variable Costs Examples

SUPPLEMENTARY PROBLEM:

Example 1.1: A company having a capacity of 1600 units per year currently is operating
at a sales level of only 1200 units, with a selling price of ₱720 per unit. The fixed costs
of the plant are ₱365,000 per year, and the variable costs are ₱416 per unit. It has been
estimated that a reduction of ₱50 per unit in the selling price would increase sales by
300 units per year.

(a.) Would this be a good program to follow?


(b.) An alternative being considered is to engage in a modernization plan that would
increase the fixed costs by ₱58,000 per year, but that would reduce the variable
costs by ₱56 per unit. Would this be a better procedure than the price-reduction
program?
(c.) Can you suggest any other program that might be superior to the foregoing?

17
College of Engineering Education
2nd Floor, B&E Building
Matina Campus, Davao City
Telefax: (082) 296-1084
Phone No.: (082)300-5456/300-0647 Local 133

Solution:

(a.) Present revenue = 720 x 1200 = 864,000

Present costs:
Fixed costs = 365,000
Variable costs = 416 x 1200 = 499200
Total = 864200
Loss = -200

New Revenue = 670 x 1500 = 1,005,000


Costs:
Fixed = 365,000
Variable = 416 x 1500 = 624000
Total = 989000
Profit = 16000
Thus, reducing the price would be a profitable program

(b.) Revenue = 864000


Costs:
Fixed = 423000
Variable = 360 x 1200 = 432000
Total = 855,000
Profit = 9000
Thus, modernization would be profitable, but it would not be as good as
procedure as the price reduction

(c.) Both programs should be combined

Self-Help: You can also refer to the sources below to help you and guide you
further understand the lesson.

Park, Chan S. (2011), Contemporary Engineering Economics (5th Edition), New Jersey:
Pearson Education, Chapters 8, 9, 11
Blank, L T., (2012), Engineering Economy (7th Edition), New York: McGraw Hill,
Chapter 9, 10, 11, 12, 13, 16, 17
Sta. Maria, H., Engineering Economy (3th Edition), National Book Store
Panneerselvam, R., (2012), Engineering Economics (Eastern Economy Edition), New
Delhi: PHI Learning Private Limited
Lindeburg, M., (2014), Civil Engineering Reference Manual for the PE Exam (14th
Edition), California: Professional Publications, Inc.

18
College of Engineering Education
2nd Floor, B&E Building
Matina Campus, Davao City
Telefax: (082) 296-1084
Phone No.: (082)300-5456/300-0647 Local 133

Let’s Check

Activity 1: Now that you have read the basic concepts in engineering economy, let us
try to check your understanding by solving the following cost problems:

1. It is the analysis and evaluation of the factors that will affect the economic success
of engineering projects to the end that a recommendation can be made which will
insure the best use of capital.
(a) Ceteris Paribus Assumption
(b) Engineering Economy
(c) Law of Demand and Supply

2. It is the quantity of certain commodity that is bought at a certain price at a given


place and time.
(a) Necessities
(b) Supply
(c) Demand

3. These refer to the to those products or services that are required to support human
life and activities, which will be purchased in somewhat the same quantity even
though the price varies considerably.
(a) Necessities
(b) Supply
(c) Demand

4. These refer to those products or services that are directly used by people to satisfy
their wants.
(a) Consumer Goods and Services
(b) Producer Goods and Services
(c) Human Goods and Services

5. These refer to those products and services that re desired by the humans and will
be purchased if money is available after the required necessities have been
obtained.
(a) Wants
(b) Needs
(c) Products

6. This refers to the total sum of the fixed cost and variable cost.
(a) Profit
(b) Revenue
(c) Total Cost

19
College of Engineering Education
2nd Floor, B&E Building
Matina Campus, Davao City
Telefax: (082) 296-1084
Phone No.: (082)300-5456/300-0647 Local 133

7. It is a quantity of certain commodity that is offered for sale at a certain price at a


given place and time.
(a) Demand
(b) Supply
(c) Needs

8. It is the capital required for most of the activities in the acquisition phase.
(a) Total Cost
(b) Investment Cost
(c) Opportunity Cost

9. It is a type of demand that occurs when a decrease in selling price result in a greater
than proportionate increase in sales.
(a) Elastic Demand
(b) Inelastic Demand
(c) Unitary Elasticity of Demand

10. It is the application of economic techniques to the evaluation of design and


engineering alternatives. It involves the systematic evaluation with the economic
merits of proposed solutions to the engineering problems.
(a) Engineering Economy
(b) Engineering Economics
(c) Engineering Management

Let’s Analyze

Activity 1: Getting acquainted with the essential terminology and basic concepts in the
study of engineering economy is not enough, one should be able to analyze a given
situation and elaborate his/her thoughts.

1. The needs and wants of people are the reasons why they are striving to earn a living
and why there’s a demand to be fulfilled. Elaborate the price-demand relationship
for luxuries and necessities of humans.
_______________________________________________________________________________________________________________
_______________________________________________________________________________________________________________
_______________________________________________________________________________________________________________
_______________________________________________________________________________________________________________
_______________________________________________________________________________________________________________
_______________________________________________________________________________________________________________

20
College of Engineering Education
2nd Floor, B&E Building
Matina Campus, Davao City
Telefax: (082) 296-1084
Phone No.: (082)300-5456/300-0647 Local 133

_________________________________________________________________________________________________________________
_________________________________________________________________________________________________________________
_________________________________________________________________________________________________________________
_________________________________________________________________________________________________________________
_________________________________________________________________________________________________________________
_________________________________________________________________________________________________________________

2. In economics, we have three types of competition namely, the Perfect Competition,


Monopoly, and Oligopoly. Do a little research about these three on how they differ and
the unique characteristics they have. Site an example for each competition.

_____________________________________________________________________
_____________________________________________________________________
_____________________________________________________________________
_____________________________________________________________________
_____________________________________________________________________
_____________________________________________________________________
_____________________________________________________________________
_____________________________________________________________________
_____________________________________________________________________
_____________________________________________________________________
_____________________________________________________________________
_____________________________________________________________________
_____________________________________________________________________
_____________________________________________________________________
_____________________________________________________________________

3. Demand is defined as the quantity of certain commodity that is purchased at a specific


time and place, similarly, supply is offered for sale. Explain further the relationship
between demand, price, and supply.
_____________________________________________________________________
_____________________________________________________________________
_____________________________________________________________________

21
College of Engineering Education
2nd Floor, B&E Building
Matina Campus, Davao City
Telefax: (082) 296-1084
Phone No.: (082)300-5456/300-0647 Local 133

_____________________________________________________________________
_____________________________________________________________________
_____________________________________________________________________
_____________________________________________________________________
_____________________________________________________________________
_____________________________________________________________________
_____________________________________________________________________
_____________________________________________________________________
_____________________________________________________________________
_____________________________________________________________________
_____________________________________________________________________

In a Nutshell

Activity 1: Based from the definition of the most essential terms in the study of
curriculum and the learning exercises that you have done, please feel free to write your
arguments or lessons learned below. I have indicated my arguments or lessons learned.

1. A demand curve or a supply curve is a relationship between two, and only two,
variables: quantity on the horizontal axis and price on the vertical axis.
2. Both direct labor and direct material costs are given on a per-unit or per-item
basis.

Your Turn

3. _______________________________________________________________
_______________________________________________________________
_______________________________________________________________
_______________________________________________________________
4. _______________________________________________________________
_______________________________________________________________
_______________________________________________________________
_______________________________________________________________

22
College of Engineering Education
2nd Floor, B&E Building
Matina Campus, Davao City
Telefax: (082) 296-1084
Phone No.: (082)300-5456/300-0647 Local 133

5. _______________________________________________________________
_______________________________________________________________
_______________________________________________________________
_______________________________________________________________
_______________________________________________________________

Q & A List

List 1: After series of discussion and exercises, kindly list down your questions or issues
in conjunction with the topics. Write the answers after clarification under the column
provided. This portion helps you in the review of concepts and essential knowledge.

Do you have any questions for clarification?


Questions/Issues Answers
1.
2.
3.
4.
5.

Keywords Index
Index: The keywords presented here will help you remember the concepts and essential
knowledge. Here are the few subject indices.

A C D F
Administrative Ex. Cost Direct Material Factory
Cost of Miscellaneous Costs Cost
Ceteris Paribus Direct Labor Costs Fixed Costs
Consumer Depreciation
Cleanup Expenses Expense
Ceilings Demand
Development Costs

G J I L

23
College of Engineering Education
2nd Floor, B&E Building
Matina Campus, Davao City
Telefax: (082) 296-1084
Phone No.: (082)300-5456/300-0647 Local 133

General Expenses Janitorial Service Indirect Labor


General Burden Expense Manufacturing Costs
Goods Expenses (IME)
Indirect Material
Interest on Loans
Insurance
Income Taxes
Incremental Costs
M N O P
Marketing Non-Quantifiable Operating Costs Prime
Expenses Factors Overhead Costs Costs
Marketing Costs Property
Microeconomics Payroll
Macroeconomics Price
Maintenance Costs Price
Floors
Producers

R S T U
Research Costs Selling Expenses Total Cost Utility costs
Rent Selling Costs Tooling Expense
Setup Expenses Tear-Down
Supervision Costs Expenses
Services Taxes
Supply
Surplus
V
Variable costs

24
College of Engineering Education
2nd Floor, B&E Building
Matina Campus, Davao City
Telefax: (082) 296-1084
Phone No.: (082)300-5456/300-0647 Local 133

Big Picture in Focus:


ULO-1b. Be able to determine the simple interest and compound interest,
cash flow diagrams and their applications to real life situations

Metalanguage
You can refer to this section, and the previous one, anytime if you run across
words which you are not familiar with. This will be a common reference in terms of the
definition of mathematical concepts that we are going to discuss.

1. Capital. This refers to the wealth in the form of money or property that can be used
to produce more wealth.

2. Interest. It is the charge paid for the use of borrowed capital or the income produced
by money which has been loaned. It can be referred to as the Cost of Money, which
is determined by an interest rate.
2.1 From the viewpoint of the lender, interest is the income produced by the
money which has been borrowed or invested. On the other hand, interest is the
amount of money paid for the use of borrowed capital from the borrower’s
perspective.

3. Discount. It is a deduction from the usual cost of products or services, typically


given from prompt or advance payment. Discount is an interest on loaned amount
immediately deducted from the loan upon release of it considering a year end period.
It is an interest paid in advance.

4. Inflation. It is the increase in the prices for goods and services from one year to
another thus decreasing the purchasing power of the money.

5. Cash Flow Diagram. This refers to a graphical representation of cash flows on a


time scale. Receipt is taken as the positive cash flow and disbursement as a
negative cash flow. Cash flow diagram can be viewed in two different perspective:
By the investor and by the borrower.

Essential Knowledge
In ULO-1b, you will be introduced to the concepts of cash flow diagram analysis,
interest, inflation, discount and how they connect from one another. At the end of this
ULO, you are expected to draw a cash flow diagram from a given situation, solve for the
interest, inflation rate, and discount. Take note that you may ask questions to your
course coordinator for clarification regarding the topic.

1. Cash Flow/Cash Flow Diagram. It is the stream of monetary values -- costs and
benefits – resulting from a project investment. Solving problems without actually
seeing it would be a very difficult task. One approach that will make economic

25
College of Engineering Education
2nd Floor, B&E Building
Matina Campus, Davao City
Telefax: (082) 296-1084
Phone No.: (082)300-5456/300-0647 Local 133

analysis easier is to illustrate it in a picture using the so-called Cash Flow Diagram.
Cash flow diagram is a graphical representation of the inflow and outflow of money
from a certain perspective or viewpoint. It is analogous to the free body diagram of
mechanical problems. It has three (3) elements:

a. A horizontal line represents time with progression of time moving from left to right.
The period labels can be applied to intervals of time rather than to point on the time
scale. A time interval is divided into an appropriate number of equal periods.
b. Arrows represent cash flow and are place at the specified period. If distinctions
are needed to be made, downward arrows represent cash outflows (expenditures,
disbursements) and upward arrows represents cash inflows (income).

Receipt (positive cash Disbursement (negative


flow or cash inflow) cash flow or cash outflow)

c. Depends on the person’s viewpoint. Unless otherwise indicated, all such cash
flows are considered to occur at the end of their respective periods. The following
symbols nomenclatures will be used:

P = present sum of money


F = future sum of money
n = number of interest periods
i = interest rate per period

Example 1b.1: A loan of ₱100 at simple interest of 10% will become ₱150 after 5 years.
F= ₱ 150

i = 10%
n= 5 years
P= ₱ 100

This would be the cash flow This would be the cash flow
diagram of the given situation on diagram of the given situation
the viewpoint of the lender: from the borrower’s perspective.

26
College of Engineering Education
2nd Floor, B&E Building
Matina Campus, Davao City
Telefax: (082) 296-1084
Phone No.: (082)300-5456/300-0647 Local 133

2. Simple Interest. It is a quick method of calculating interest charged on a loan. It is


determined by multiplying the interest rate by the principal by the number of periods.
These type of interest benefits consumers who pay loans on time or early each
month. Auto loans and short-term personal loans are common examples of simple
interest loans.

The general formula in solving the future amount after “n” periods is expressed
as follows:

𝐼 = 𝑃𝑛𝑖 (1b.1)

𝐹 =𝑃+𝐼 (1b.2)

𝐹 = 𝑃(1 + 𝑛𝑖) (1b.3)


Where:
I = interest
P = principal or present worth
n= time in years or fraction of a year
i = rate of interest per interest period
F = accumulated amount or future worth
Example 1b.2: What will be the future worth of money after 14months, if a sum of
P10,000 is invested today at a simple interest rate of 12% per year?

Given: 𝑃 = ₱10,000 Solution:


𝑖 = 12%
𝑛 = 14 𝑚𝑜𝑛𝑡ℎ𝑠
14
𝐹 = 𝑃( 1 + 𝑛𝑖 ) = (10,000) (1 + ( ) (0.12))
Required: 𝐹 =? 12
= ₱𝟏𝟏, 𝟒𝟎𝟎

Two types of Simple Interest:


a. Ordinary Simple Interest (OSI). It is based on 30 days per month or 360 days
per year (also known as the banker’s year)

𝐷
𝐼𝑂𝑆𝐼 = Pi (1b.4)
360
Where:
I = interest i = rate of interest per interest period
P = principal or present worth D = number of days

27
College of Engineering Education
2nd Floor, B&E Building
Matina Campus, Davao City
Telefax: (082) 296-1084
Phone No.: (082)300-5456/300-0647 Local 133

Example 1b.3: Find the future amount and the ordinary simple interest on ₱1,000 for 8
months and 20 days at an interest rate of 10%.

Given: 𝑃 = ₱1,000 Solution:


𝑖 = 10% 30
Number of days= 8 𝑚𝑜𝑠. ( ) + 20 = 260
𝐷 = 8 𝑚𝑜𝑛𝑡ℎ𝑠 𝑎𝑛𝑑 20 𝑑𝑎𝑦𝑠 1 𝑚𝑜.

Required: 𝐹 =? 𝐷 260
𝐼𝑂𝑆𝐼 = 𝑃𝑖 ( ) = (1000)(0.1) ( ) = 72.22
360 360

𝐹 = 𝑃 + 𝐼 = 1000 + 72.22 = ₱𝟏, 𝟎𝟕𝟐. 𝟐𝟐

1.2 Exact Simple Interest (ESI). It is based on the exact number of days in a year, 365
days for ordinary year or 366 days for a leap year.

𝐷 𝐷
𝐼 = Pr or 𝐼 = Pr (1b.5)
365 366
Where:
I = interest
P = principal or present worth
i = rate of interest per interest period
D = number of days
Note: A year is a leap year if it is divisible by 4 provided it doesn’t end with ’00. If it ends
with ’00, it is only a leap year when it is divisible by 400.

LEAP YEAR MATRIX YEAR NOT ENDING WITH '00


1989 1993 1997 2001 2005 2009 2013 2017
1990 1994 1998 2002 2006 2010 2014 2018
1991 1995 1999 2003 2007 2011 2015 2019
1992 1996 2000 2004 2008 2012 2016 2020
CENTURY YEAR
1300 1700 2100 2500 2900 3300 3700 4100
1400 1800 2200 2600 3000 3400 3800 4200
1500 1900 2300 2700 3100 3500 3900 4300
1600 2000 2400 2800 3200 3600 4000 4400

28
College of Engineering Education
2nd Floor, B&E Building
Matina Campus, Davao City
Telefax: (082) 296-1084
Phone No.: (082)300-5456/300-0647 Local 133

Example 1b.4: Determine the exact simple interest on P500 for the period from January
10 to October 28, 1996 at 16% interest.
Given: 𝑃 = 500 Required: 𝐼𝐸𝑆𝐼 =?
𝑖 = 16%
𝐷 = 𝐽𝑎𝑛𝑢𝑎𝑟𝑦 10 𝑡𝑜 𝑂𝑐𝑡𝑜𝑏𝑒𝑟 28, 1996
Solution:
𝐷 292
Jan 10-31 - 21 (excluding Jan 10) 𝐼𝐸𝑆𝐼 = 𝑃𝑖 ( ) = (500)(0.16) ( )
February - 29 366 366
March - 31 = ₱ 𝟔𝟑. 𝟖𝟑
April - 30
May - 31
June - 30
July - 31
August - 31
September - 30
October - 28 (including Oct.28)
292 days

3. Compound Interest. Compound interest is an interest on top of interest. The


general formula in solving the future amount after “n” periods is expressed as
follows:

F = P(1 + i)n (From Present to Future) (1b.6)

P = F(1 + i)−n (From Future to Present) (1b.7)

Where:
F = future worth
P = principal or present worth
i = effective rate of interest per interest period
n= total number of compounding periods
Single Payment Compound Amount Factor, F/P:

𝐹⁄ = (1 + i)n = (𝐹⁄ , 𝑖%, 𝑛) (1b.8)


𝑃 𝑃
Single Payment Present Worth Factor, P/F:

𝑃⁄ = (1 + i)−n = (𝑃⁄ , 𝑖%, 𝑛) (1b.9)


𝐹 𝐹

29
College of Engineering Education
2nd Floor, B&E Building
Matina Campus, Davao City
Telefax: (082) 296-1084
Phone No.: (082)300-5456/300-0647 Local 133

Cash Flow Diagram of Compound Interest from the Borrower’s Viewpoint

Rate of Interest
In solving for the compound interest, we may encounter the different types of
interest rate namely the effective interest rate and the nominal interest rate. You may
get confused when dealing with either of the two, thus study the following carefully.

(a) Nominal Rate of Interest. This specifies the rate of interest and the number of
interest periods in one year. It is also called the stated interest rate since this
interest works according to the simple interest and does not consider the
compounding periods.
Use equation 1b.10 to find the rate of interest per interest period given the
nominal interest rate, r, and the number of compounding periods per year.

𝑟
𝑖= (1b.10)
𝑚
Where:
i = rate of interest per interest period
r = nominal rate of interest
m = number of compounding per year

Example 1b.5: If the nominal interest rate is 10% compounded quarterly, what is its
rate of interest per interest period?

Given: Solution:
𝑟 = 10% = 0.1 𝑟 0.1
𝑚 = 𝑐𝑜𝑚𝑝𝑜𝑢𝑛𝑑𝑒𝑑 𝑞𝑢𝑎𝑟𝑡𝑒𝑟𝑙𝑦 = 4 𝑖= = = 𝟎. 𝟎𝟐𝟓 𝒐𝒓 𝟐. 𝟓%
𝑚 4
Required:
𝑖 =?

30
College of Engineering Education
2nd Floor, B&E Building
Matina Campus, Davao City
Telefax: (082) 296-1084
Phone No.: (082)300-5456/300-0647 Local 133

Example 1b.6: Compute for the values of 𝑖 and 𝑛, if the nominal interest rate is 15%
and the number of years of investment is 5.

Given:
𝑟 = 15% = 0.15
𝑛𝑢𝑚𝑏𝑒𝑟 𝑜𝑓 𝑦𝑒𝑎𝑟𝑠 𝑜𝑓 𝑖𝑛𝑣𝑒𝑠𝑡𝑚𝑒𝑛𝑡 (𝑡) = 5

(a) compounded annually (𝑚 = 1) (d) compounded monthly (𝑚 = 12)


𝑟 0.15
𝒊= = = 𝟎. 𝟏𝟓 𝑟 0.15
𝑚 1 𝒊= = = 𝟎. 𝟎𝟏𝟐𝟓
𝑚 12
𝒏 = (𝑡)(𝑚) = (5)(1) = 𝟓
𝒏 = (𝑡)(𝑚) = (5)(12) = 𝟔𝟎
(b) compounded semi-annually (𝑚 = 2)
(e) compounded bi-monthly (𝑚 = 6)
𝑟 0.15
𝒊= = = 𝟎. 𝟎𝟕𝟓 𝑟 0.15
𝑚 2 𝒊= = = 𝟎. 𝟎𝟐𝟓
𝑚 6
𝒏 = (𝑡)(𝑚) = (5)(2) = 𝟏𝟎
𝒏 = (𝑡)(𝑚) = (5)(6) = 𝟑𝟎
(c) compounded quarterly (𝑚 = 4)
𝑟 0.15
𝒊= = = 𝟎. 𝟎𝟑𝟕𝟓
𝑚 4

𝒏 = (𝑡)(𝑚) = (5)(4) = 𝟐𝟎

(b) Effective Rate of Interest. This refers to the exact or actual rate of interest on
the principal (P) for one year. It caters the compounding periods during a payment
plan and is used to compare the annual interest between loans with different
compounding periods like week, month, year, etc. It should be noted that the
effective interest rates are always expressed on an annual basis. It is defined by
the equation:

𝐸𝑅 = 𝐹1 − 1 = (1 + 𝑖)𝑚 − 1 (1b.11)

Where:
F = future worth
i = rate of interest per interest period
m = number of compounding per year

31
College of Engineering Education
2nd Floor, B&E Building
Matina Campus, Davao City
Telefax: (082) 296-1084
Phone No.: (082)300-5456/300-0647 Local 133

Example 1b.7: If ₱ 1.00 is invested at a nominal rate of 15% compounded quarterly,


what is its effective rate of interest?

Given: Solution:
𝑟 = 15% = 0.15
𝑚=4 𝑚
0.15 4
𝐸𝑅 = (1 + 𝑖) − 1 = (1 + ) −1
4
Required: = 𝟎. 𝟏𝟓𝟖𝟕 or 𝟏𝟓. 𝟖𝟕%
𝐸𝑅 =?

Example 1b.8: Find the nominal rate which, if converted quarterly, could be used
instead of 12% compounded monthly. What is the corresponding effective rate?

Given: Required:
𝑚1 = 4 𝑟1 =?
𝑟2 = 12% = 0.12 𝐸𝑅 =?
𝑚2 = 12

Solution:
To make two or more nominal rates equivalent, their corresponding effective rates must
be equal. Thus, we should equate the two by formulating:

𝐸𝑅1 = 𝐸𝑅2
𝑟1 𝑚1 𝑟2 𝑚2
(1 + ) − 1 = (1 + ) −1
𝑚1 𝑚2
𝑟1 4 0.12 12
(1 + ) − 1 = (1 + ) −1
4 12
𝑟1 4
(1 + ) = (1.01)12 − 1 + 1
4
𝑟1
1 + = (1.01)3
4
𝑟1
= 1.0303 − 1
4
𝑟1 = (0.0303)(4)

𝑟1 = 𝟎. 𝟏𝟐𝟏𝟐 or 𝟏𝟐. 𝟏𝟐%

𝐸𝑅 = 𝟏. 𝟎𝟑𝟎𝟑

32
College of Engineering Education
2nd Floor, B&E Building
Matina Campus, Davao City
Telefax: (082) 296-1084
Phone No.: (082)300-5456/300-0647 Local 133

Note: For several compounding periods per year in a specific scenario, the calculation
for compound interest is as follows:

𝐹 = 𝑃[(1 + 𝑖 )𝑛𝑎𝑛𝑛𝑢𝑎𝑙𝑙𝑦 (1 + 𝑖 )𝑛𝑞𝑢𝑎𝑟𝑡𝑒𝑟𝑙𝑦 (1 + 𝑖 )𝑛𝑚𝑜𝑛𝑡ℎ𝑙𝑦 … ]

4. Continuous Compound Interest

𝐹 = 𝑃𝑒 𝑟𝑡 (1b.12)

𝑃 = 𝐹𝑒 −𝑟𝑡 (1b.13)

Where:
F = future worth
P = present worth
r = nominal rate of interest
t = number of years

Example 1b.9: ₱100, 000 is deposited in a bank that earns 5% compounded


continuously. What will be the amount after 10 years?

Given: Required:
𝑃 = ₱100,00 𝐹 =?
𝑟 = 5% = 0.05
𝑡 = 10
Solution:
𝐹 = 𝑃𝑒 𝑟𝑡
𝐹 = (100,000)𝑒 (0.05)(10)
𝐹 = ₱𝟏𝟔𝟒, 𝟖𝟕𝟐. 𝟏𝟑

Example 1b.10: Money is deposited in a certain account for which interest is


compounded continuously. If the balance doubles in 6 years, what is the annual
percentage rate?

Given: Solution:
𝐹 = 2𝑃
𝐹 = 𝑃𝑒 𝑟𝑡
𝑡=6
Required: 2𝑃 = 𝑃𝑒 𝑟(6)
𝑟 =? 𝑟 = 𝟎. 𝟏𝟏𝟓𝟓 or 𝟏𝟏. 𝟓𝟓%
5. Discount. It is a deduction from the usual cost of products or services, typically
given from prompt or advance payment. Discount is an interest on loaned amount

33
College of Engineering Education
2nd Floor, B&E Building
Matina Campus, Davao City
Telefax: (082) 296-1084
Phone No.: (082)300-5456/300-0647 Local 133

immediately deducted from the loan upon release of it considering a year end period.
It is an interest paid in advance.

Discount can be simply defined as the difference between the future worth of the
commodity and its present worth.

𝐷 =𝐹−𝑃 (1b.14)

Where:
D = amount of discount
F = future worth
P = present worth
There are times that problems concerning discounts have different givens. Here are
more helpful equations in solving discount problems.

Discount rate, denoted by d, is the discount on one unit of principal per unit of time.
This is given by the formula:
𝐹−𝑃
𝑑= = 1 − (1 + 𝑖)−1 (1b.15)
𝐹
If the problem states that the commodity is discounted in a certain period of time,
then use the following equations:

𝑃 = 𝐹(1 − 𝑛𝑑) (1b.16)

The next equations show the relationship between discount rate and interest rate:

𝑑 (1b.17)
𝑖=
1−𝑑
𝑖
𝑑= (1b.18)
1+𝑖
Where:
d = discount rate for the period involved
i = interest rate for the same period

34
College of Engineering Education
2nd Floor, B&E Building
Matina Campus, Davao City
Telefax: (082) 296-1084
Phone No.: (082)300-5456/300-0647 Local 133

Example 1b.11: Mr. Lee borrowed money from the bank. He receives ₱ 1, 340 and
promised to pay ₱ 1, 500 at the end of 9 months. Compute for its (a) simple interest
rate; and (b) discount rate.

Given: Solution:
𝑃 = ₱1, 340 (a) 𝐹 = 𝑃(1 + 𝑛𝑖)
𝐹 = ₱1, 500 9
𝑛 = 9 𝑚𝑜𝑛𝑡ℎ𝑠 1, 500 = 1, 340 (1 + 𝑖)
12
Required: 𝑖 = 𝟎. 𝟏𝟓𝟗𝟐 or 𝟏𝟓. 𝟗𝟐%
𝑖 =?
𝑑 =? 𝑖
(b) 𝑑 =
1+𝑖
0.1592
𝑑= 1+0.1592
= 𝟎. 𝟏𝟑𝟕𝟑 or 𝟏𝟑. 𝟕𝟑%

6. Inflation. Inflation is the increase in the prices for goods and services from one year
to another thus decreasing the purchasing power of the money.

𝐹𝐶 = 𝑃𝐶 (1 + 𝑓)𝑛 (1b.19)

Where:
FC = future cost of a commodity
PC = present cost of the same commodity
f = inflation rate
n = number of years

In an inflationary economy, the buying power of money decreases as costs increases:

Case 1: If interest is not compounded at the time inflation is occurring


𝑃
𝐹= (1b.20)
(1 + 𝑓 )𝑛

Case 2: If interest is being compounded at the same time that inflation is occurring

𝑃 (1 + 𝑖 )𝑛 1+𝑖 𝑛
𝐹= = 𝑃( ) (1b.21)
(1 + 𝑓 ) 𝑛 1+𝑓

*If RC is not specified, use RC=FC


Where:
F = Future worth measured in today’s present amount P
P = Present amount
f = inflation rate
n = number of years

35
College of Engineering Education
2nd Floor, B&E Building
Matina Campus, Davao City
Telefax: (082) 296-1084
Phone No.: (082)300-5456/300-0647 Local 133

i = rate of interest
Example 1b.12: A man invested ₱10,000 at an interest rate of 10% compounded
annually. What will be the final amount of his investment, in terms of today’s pesos,
after five years, if inflation remains the same at the rate of 8% per year?

Given:
𝑃 = ₱10, 000
𝑖 = 10%
𝑓 = 8%
𝑛=5

Required:
𝐹 =?

Solution:
1+𝑖 𝑛 1 + 0.10 5
𝐹 = 𝑃( ) = ₱10,000 ( ) = ₱𝟏𝟎𝟗𝟔𝟎. 𝟖𝟔
1+𝑓 1 + 0.08

Self-Help: You can also refer to the sources below to help you and guide you
further understand the lesson

Park, Chan S. (2011), Contemporary Engineering Economics (5th Edition), New Jersey:
Pearson Education, Chapters 8, 9, 11
Blank, L T., (2012), Engineering Economy (7th Edition), New York: McGraw Hill,
Chapter 9, 10, 11, 12, 13, 16, 17
Sta. Maria, H., Engineering Economy (3th Edition), National Book Store
Panneerselvam, R., (2012), Engineering Economics (Eastern Economy Edition), New
Delhi: PHI Learning Private Limited
Lindeburg, M., (2014), Civil Engineering Reference Manual for the PE Exam (14th
Edition), California: Professional Publications, Inc.

36
College of Engineering Education
2nd Floor, B&E Building
Matina Campus, Davao City
Telefax: (082) 296-1084
Phone No.: (082)300-5456/300-0647 Local 133

Let’s Check

Activity 1: Now that we are done with the basic concepts of economic analysis, it is
your turn to showcase the things you have learned. Solve the following problems and
show clear solutions.

1. How many years will it take for an investment to double, if the interest rate is 8% per
year, compounded annually?

2. How much money will be required four years from today to repay a ₱2,000 loan that
is made today at 8% interest compounded annually?

3. How much money will be required four years from today to repay a ₱2,000 loan that
is made today at 8% simple interest?

4. At what rate of interest, compounded annually will an investment triple itself in 8


years?

5. If ₱300 is earned in three months on an investment of ₱12,000, what is the annual


rate of simple interest?

Let’s Analyze

Activity 1: Solve the following problems and show your solutions clearly.

1. On the first day of the year, a man deposits ₱1,000 in a bank at 8% per year
compounded annually. He withdraws ₱80 at the end of the first year, ₱90 at the end of
the second year, and the remaining balance at the end of the third year. How much
does he withdraw at the end of the third year?

2. What amount of money is equivalent to receiving ₱5,000 two years from today, if
interest is compounded quarterly at the rate of 2.5% per quarter?

3. On the first day of the year, a man deposits ₱1,000 in a bank at 8% per year
compounded annually. He withdraws ₱80 at the end of the first year, ₱90 at the end of
the second year, and the remaining balance at the end of the third year. How much
better off, in terms of net cash flow, would he have been if he had not made the
withdrawals at the ends of years one and two?

4. A person lends ₱2,000 for five years at 10% per annum simple interest; then the
entire proceeds are invested for 10 years at 9% per year, compounded annually. How
much money will the person have at the end of the entire 15-year period?

37
College of Engineering Education
2nd Floor, B&E Building
Matina Campus, Davao City
Telefax: (082) 296-1084
Phone No.: (082)300-5456/300-0647 Local 133

5. On the first day of the year, a man deposits ₱1,000 in a bank at 8% per year
compounded annually. He withdraws ₱80 at the end of the first year, ₱90 at the end of
the second year, and the remaining balance at the end of the third year. What is the net
cash flow?

In a Nutshell

Activity 1: Based from the definition of the most essential terms in the study of
curriculum and the learning exercises that you have done, please feel free to write your
arguments or lessons learned below. I have indicated my arguments or lessons learned.

1. There are two types of simple interest; ordinary and exact simple interest.
2. Ordinary simple interest is based on 360 banker days per year.

Your Turn

3. _______________________________________________________________
_______________________________________________________________
_______________________________________________________________
_______________________________________________________________
_______________________________________________________________
4. _______________________________________________________________
_______________________________________________________________
_______________________________________________________________
_______________________________________________________________
_______________________________________________________________
5. _______________________________________________________________
_______________________________________________________________
_______________________________________________________________
_______________________________________________________________
_______________________________________________________________

38
College of Engineering Education
2nd Floor, B&E Building
Matina Campus, Davao City
Telefax: (082) 296-1084
Phone No.: (082)300-5456/300-0647 Local 133

Q & A List

List 1: After series of discussion and exercises, kindly list down your questions or issues
in conjunction with the topics. Write the answers after clarification under the column
provided. This portion helps you in the review of concepts and essential knowledge.

Do you have any questions for clarification?

Questions/Issues Answers

1.

2.

3.

4.

5.

Keywords Index

Index: The keywords presented here will help you remember the concepts and essential
knowledge. Here are the few subject indices.

C D E I
Compound Discount Exact simple Interest
interest interest Inflation
Effective interest
rate
N O R S
Nominal interest Ordinary simple Rate of return Simple Interest
rate interest

39
College of Engineering Education
2nd Floor, B&E Building
Matina Campus, Davao City
Telefax: (082) 296-1084
Phone No.: (082)300-5456/300-0647 Local 133

Big Picture
Week 4-5: Unit Learning Outcomes-Unit 2 (ULO-2): At the end of the unit, you are
expected to:

a. Be able to apply the concept of annuities in the decision-making process


through the present economic studies; and
b. Solve for the uniform gradient amount of the given real-life economic
situations.

Big Picture in Focus:


ULO-2a. Be able to apply the concept of annuities in the decision-making
process through the present economic studies.

Metalanguage
This section will be your reference regarding some terms related to the concept
of annuities. This will serve as our basis in dealing with the terms we are unfamiliar with
or the words we are not quite sure of its meaning.

1. Annuity. An annuity is a series of payments made at equal periods or equal end of


year payments. This may occur in the payment of debt, accumulation, substitution
of series of equal amounts and other financial activities such as installment
payments, monthly rentals, life insurance premium and monthly retirement benefits.
Essential Knowledge
In ULO-2a, you will run across numerous equations dealing with the concept of
annuity which you need to comprehend and familiarize since this will be essential in
economic analysis. It is recommended, in this unit, to practice solving problems
involving annuities so that it will not be too difficult for you to understand the given
situations.
1. Annuity. It is divided into five (5) major categories. Each of which has its unique
characteristic that you need to be acquainted with.

1.1. Ordinary Annuity – it is a type of annuity where equal payments are made at
the end of each period.

𝐴 −𝑛 ]
𝐴 (1 + 𝑖)𝑛 − 1
𝑃= [ (
1− 1+𝑖 ) = [ ] (2a.1)
𝑖 𝑖 (1 + 𝑖)𝑛

40
College of Engineering Education
2nd Floor, B&E Building
Matina Campus, Davao City
Telefax: (082) 296-1084
Phone No.: (082)300-5456/300-0647 Local 133

Uniform Series Present Worth Factor (𝑃⁄𝐴 , 𝑖%, 𝑛 ):

𝑛
𝑃⁄ = [(1 + 𝑖) − 1] (2a.2)
𝐴 𝑖(1 + 𝑖)𝑛

Capital Recovery Factor (𝐴⁄𝑃 , 𝑖%, 𝑛 ):

𝑛
𝐴⁄ = [ 𝑖(1 + 𝑖) ]
𝑃 (1 + 𝑖)𝑛 − 1 (2a.3)

Where:
P = value or sum of money at present
A = series of periodic equal amount of payments
i = interest rate per interest period
n = number of interest periods/number of equal payments
P/A = uniform series present worth factor
A/P = capital recovery factor

Example 2a.1: Find the annual payment to extinguish a debt of ₱100,000 payable for
6 years at 12% interest annually.

Given: Solution:
𝑃 = ₱100,000 𝐴
𝑃 = [1 − (1 + 𝑖)−𝑛 ]
𝑛 = 6 𝑦𝑒𝑎𝑟𝑠 𝑖
𝑖 = 12% 𝑜𝑟 0.12 𝐴
₱100,000 = [1 − (1 + 0.12)−6 ]
Required: 0.12
𝐴 =? 𝐴 = ₱ 𝟐𝟒, 𝟑𝟐𝟐. 𝟓𝟕

For calculating the Future Worth given the annuity and interest rate, use the formula:

𝐴
𝐹= [(1 + 𝑖 )𝑛 − 1] (2a.4)
𝑖

Uniform Series Compound Amount Factor (𝐹⁄𝐴 , 𝑖%, 𝑛):

𝑛
𝐹⁄ = [(1 + 𝑖) − 1] (2a.5)
𝐴 𝑖

41
College of Engineering Education
2nd Floor, B&E Building
Matina Campus, Davao City
Telefax: (082) 296-1084
Phone No.: (082)300-5456/300-0647 Local 133

Sinking Fund Factor (𝐴⁄𝐹 , 𝑖%, 𝑛):

𝐴⁄ = [ 𝑖
𝐹 ] (2a.6)
(1 + 𝑖)𝑛 − 1

Where:
F = value or sum of money at some future time
A = series of periodic equal amount of payments
i = interest rate per interest period
n = number of interest periods/number of equal payments
F/A = uniform series compound amount factor
A/F = sinking fund factor

Example 2a.2: What annuity is required over 12 years to equate to a future amount of
₱200,000 when i=8%?

Given: Solution:
𝐹 = ₱200,000 𝐴
𝐹 = 𝑖 [(1 + 𝑖)𝑛 − 1]
𝑛 = 12 𝑦𝑒𝑎𝑟𝑠
𝐴
𝑖 = 8% 𝑜𝑟 0.08 ₱200,000 = [(1 + 0.08)12 − 1]
Required: 0.08
𝐴 =? 𝐴 = ₱ 𝟏𝟎, 𝟓𝟑𝟗

Example 2a.3: An annual payment is made for 10 years with an annual interest rate of
8%. Compute the following (a) Uniform Series Present Worth Factor; (b) Capital
Recovery Factor; (c) Uniform Series Compound Amount Factor; and (d) Sinking Fund
Factor.

Given: Solution:
𝑛 = 10 𝑦𝑒𝑎𝑟𝑠
(1+𝑖)𝑛 −1 (1+.08)10−1
𝑖 = 8% 𝑜𝑟 0.08 (a) 𝑃⁄𝐴 = [ ]=[ ] = 𝟔. 𝟕𝟏
𝑖(1+𝑖)𝑛 .08(1+.08)10
Required:
(a) 𝑃/𝐴 =? 𝑖(1+𝑖)𝑛 .08(1+.08)10
(b) 𝐴/𝑃 =? (b) 𝐴⁄𝑃 = [(1+𝑖)𝑛 −1] = [(1+.08)10−1] = 𝟎. 𝟏𝟒𝟗
(c) 𝐹/𝐴 =?
(d) 𝐴/𝐹 =?

Cont’n:
(1+𝑖)𝑛 −1 (1+.08)10 −1
(c) 𝐹⁄𝐴 = [ ]=[ ] = 𝟏𝟒. 𝟒𝟖𝟕
𝑖 .08

𝑖 .08
(b) 𝐴⁄𝐹 = [(1+𝑖)𝑛 −1] = [(1+.08)10−1] = 𝟎. 𝟎𝟔𝟗

42
College of Engineering Education
2nd Floor, B&E Building
Matina Campus, Davao City
Telefax: (082) 296-1084
Phone No.: (082)300-5456/300-0647 Local 133

1.2. Annuity Due – it is a type of annuity where equal payments are made at the
beginning of each period.

𝐴 (1 + 𝑖)𝑛−1 − 1
𝑃= 𝐴+
(2a.7) [ ]
𝑖 (1 + 𝑖)𝑛−1
Where:
P = value or sum of money at present
A = series of periodic equal amount of payments
i = interest rate per interest period
n = number of interest periods/number of equal payments

Example 2a.4: A man agrees to make equal payments at the beginning of each 6
months for 10 years to discharge a debt of ₱50,000 due now. If money is worth 8%
compounded semiannually, find the semiannual payment.

Given: Solution:
𝑛 = 10 𝑦𝑒𝑎𝑟𝑠 𝑟 0.08
Find i using 𝑖 = : 𝑖= = 0.04
𝑟 = 8% 𝑜𝑟 0.08 𝑚 2
𝑚=2
Solve for A:
𝑃 = ₱50,000
𝐴 (1 + 𝑖)𝑛𝑚−1 − 1
𝑃 = 𝐴+ [ ]
Required: 𝑖 (1 + 𝑖)𝑛𝑚−1
𝐴 =?
𝐴 (1 + 0.04)(10)(2)−1 − 1
₱50,000 = 𝐴 + [ ]
0.04 (1 + 0.04)(10)(2)−1

𝐴 = ₱𝟑, 𝟓𝟑𝟕. 𝟓𝟖

43
College of Engineering Education
2nd Floor, B&E Building
Matina Campus, Davao City
Telefax: (082) 296-1084
Phone No.: (082)300-5456/300-0647 Local 133

The Future Worth of Annuity Due can be calculated by:

𝐴
[(1 + 𝑖)𝑛+1 − 1] − 𝐴
𝐹 = (2a.8)
𝑖

Where:
F = value or sum of money at some future time
A = series of periodic equal amount of payments
i = interest rate per interest period
n = number of interest periods/number of equal payments

Example 2a.5: To accumulate a fund of ₱80,000 at the end of 10 years, a man will
make equal annual deposits in the fund at the beginning of each year. How much should
he deposit if the fund is invested at 5% compounded annually?

Given: Solution:
𝐹 = ₱80,000
𝑛 = 10 𝑦𝑒𝑎𝑟𝑠 Solve for A:
𝑖 = 5% 𝑜𝑟 0.05 𝐴
𝐹= [(1 + 𝑖)𝑛+1 − 1] − 𝐴
𝑖
Required:
𝐴
𝐴 =? ₱80,000 = [(1 + 0.05)10+1 − 1] − 𝐴
0.05

𝐴 = ₱𝟔, 𝟎𝟓𝟕. 𝟒𝟗

1.3. Perpetuity – it is a type of annuity where the payment periods continue


indefinitely.

𝐴
𝑃= (2a.9)
𝑖

44
College of Engineering Education
2nd Floor, B&E Building
Matina Campus, Davao City
Telefax: (082) 296-1084
Phone No.: (082)300-5456/300-0647 Local 133

Example 2a.6: Find the present worth of perpetuity of ₱5,200 payable monthly if the
interest is 16% compounded monthly.

Given: Solution:
𝐴 = ₱5,200
𝑚 = 12 Solve for i using 𝑖 = :
𝑟
𝑖=
0.16
= 0.0133
𝑟 = 16% 𝑜𝑟 0.16 𝑚 12

Solve for P:
Required:
₱5,200
𝑃 =? 𝑃=
0.0133
𝑃 = ₱390,000

𝐴 = ₱𝟔, 𝟎𝟓𝟕. 𝟒𝟗
Example 2a.7: Find the present value of a perpetuity of ₱15,000 payable semiannually
if money is worth 8% compounded quarterly.

Given: Solution:
𝐴 = ₱15,000
𝑚1 = 2 First, solve for 𝑟1 using Effective Rate of Interest
𝑟2 = 8% 𝑜𝑟 0.08 formula:
𝑚2 = 4 𝑟1 𝑚1 𝑟2 𝑚2
(1 + ) − 1 = (1 + ) −1
𝑚1 𝑚2
Required:
𝑟1 =? 𝑟1 2 0.08 4
(1 + ) − 1 = (1 + ) −1
𝑃 =? 2 4
𝑟1 = 0.0808 𝑜𝑟 8.08%
𝑟1 0.0808
∴𝑖= = = 0.0404 𝑜𝑟 4.04%
𝑚1 2

Solve for P:
₱15,000
𝑃=
0.0404
𝑷 = ₱𝟑𝟕𝟏, 𝟐𝟖𝟕. 𝟏𝟑
1.4. Deferred Annuity – it is a type of annuity where the first payment is made
several periods after the beginning of annuity.

𝑨 (𝟏+𝒊)𝒏 −𝟏
𝑷= 𝒊[ (𝟏+𝒊)𝒏
] (𝟏 + 𝒊)−𝒎 (2a.10)
Where:
P = value or sum of money at present time
A = series of periodic equal amount of payments
i = interest rate per interest period
n = number of equal payments
m = number of interest periods with no payments
made

45
College of Engineering Education
2nd Floor, B&E Building
Matina Campus, Davao City
Telefax: (082) 296-1084
Phone No.: (082)300-5456/300-0647 Local 133

Example 2a.8: A boy is entitled to 10 yearly endowments of ₱30,000 each starting at


the end of the eleventh year from now. Using an interest rate of 8% compounded
annually, what is the value of these endowments now?

Given: Solution:
𝐴 = ₱30,000
𝑛 = 10
𝑖 = 8% 𝑜𝑟 0.08
𝑚 = 10

Required:
𝑃 =?

Solve for P:
𝐴 (1+𝑖)𝑛 −1
𝑃= [ (1+𝑖)𝑛
] (1 + 𝑖)−𝑚
𝑖
30,000 (1+0.08)10 −1
𝑃= [ (1+0.08)10 ] (1 + 0.08)−10
0.08

𝑷 = ₱𝟗𝟑, 𝟐𝟒𝟏. 𝟗𝟖
1.5. Continuous Compounding for Discrete Payments– it is a type of annuity
where payments are compounded continuously.

In computing annuity with continuous compounding, replace interest rate ( i )


of other annuity formulas with the effective rate for compounded continuously
given by the formula:

𝐸𝑅 = 𝑒 𝑟 − 1 (2a.11)

Example 2a.9: Determine the accumulated amount to an account paying ₱5,000


annually (payments are made at the beginning of each period) for 18 years if money is
worth 8% compounded continuously.

Given: Solution:
𝐴 = ₱5,000
𝑛 = 18 𝑦𝑒𝑎𝑟𝑠 Solve for F using Annuity Due formula replacing i with
𝑟 = 8% 𝑜𝑟 0.08 ER:
𝐴
𝐹 = [(1 + 𝑖)𝑛+1 − 1] − 𝐴
𝑖
Required:
𝐴
𝐹 =? 𝐹= [(1 + (𝑒 𝑟 − 1))𝑛+1 − 1] − 𝐴
𝑒 𝑟 −1

𝑭 = ₱𝟐𝟎𝟗, 𝟒𝟓𝟐. 𝟓𝟕

46
College of Engineering Education
2nd Floor, B&E Building
Matina Campus, Davao City
Telefax: (082) 296-1084
Phone No.: (082)300-5456/300-0647 Local 133

Self-Help: You can also refer to the sources below to help you and guide you
further understand the lesson

Park, Chan S. (2011), Contemporary Engineering Economics (5th Edition), New Jersey:
Pearson Education, Chapters 8, 9, 11

Blank, L T., (2012), Engineering Economy (7th Edition), New York: McGraw Hill,
Chapter 9, 10, 11, 12, 13, 16, 17

Sta. Maria, H., Engineering Economy (3th Edition), National Book Store

Panneerselvam, R., (2012), Engineering Economics (Eastern Economy Edition), New


Delhi: PHI Learning Private Limited

Lindeburg, M., (2014), Civil Engineering Reference Manual for the PE Exam (14th
Edition), California: Professional Publications, Inc.

Let’s Check

Activity 1: Now that you have reviewed the annuity problems in engineering economy,
let us try to check your understanding by solving the following:

1. A student deposits ₱1,000 in a savings account that pays interest at the rate of 6%
per year compounded annually. If all the money is allowed to accumulate, how much
money will the student have after 12 years?

2. A certain sum of money will be deposited in a savings account that pays interest at
the rate of 6% per year compounded annually. If all the money is allowed to
accumulate, how much must be deposited initially so that ₱5000 will have
accumulated after 10 years?

3. What are the present worth and the accumulated amount of a 10-year annuity paying
₱10,000 at the end of each year, with interest at 15% compounded annually?

47
College of Engineering Education
2nd Floor, B&E Building
Matina Campus, Davao City
Telefax: (082) 296-1084
Phone No.: (082)300-5456/300-0647 Local 133

Let’s Analyze

Activity 1: Solve the following problems and write the step-by-step procedure you did
to find what is asked.

1. An engineer who is planning his retirement has decided that he will have to withdraw
₱10 000 from his savings account at the end of each year. How much money must the
engineer have in the bank at the start of his retirement, if his money earns 6% per year,
compounded annually, and he is planning a 12-year retirement (say 12 annual
withdrawals)?

2. An engineer is planning for a 15-year retirement. In order to supplement his pension


and offset the anticipated effects of inflation, he intends to withdraw₱5000 at the end of
the first year, and to increase the withdrawal by ₱1000 at the end of each successive
year. How much money must the engineer have in his savings account at the start of
his retirement, if money earns 6% per year, compounded annually?

3. How much money must initially be deposited in a savings account paying 5% per
year, compounded annually, to provide for ten annual withdrawals that start at ₱6000
and decrease by ₱500 each year?

4. A man has deposited ₱50 000 in a retirement income plan with a local bank. This
bank pays 9% per year, compounded annually, on such deposits. What is the maximum
amount the man can withdraw at the end of each year and still have the funds last for
12 years?

5. A man has deposited ₱50 000 in a retirement income plan with a local bank. This
bank pays 8.75 % per year, compounded annually, on such deposits. What is the
maximum amount the man can withdraw at the end of each year and still have the funds
last for 12 years?

In a Nutshell
Activity 1: Based from the definition of the most essential terms in the study of
curriculum and the learning exercises that you have done, please feel free to write your
arguments or lessons learned below. I have indicated my arguments or lessons learned.

1. Annuity is an end-of-year equal payment.

Your Turn

2. _______________________________________________________________
_______________________________________________________________

48
College of Engineering Education
2nd Floor, B&E Building
Matina Campus, Davao City
Telefax: (082) 296-1084
Phone No.: (082)300-5456/300-0647 Local 133

_______________________________________________________________
_______________________________________________________________
3. _______________________________________________________________
_______________________________________________________________
_______________________________________________________________
_______________________________________________________________
4. _______________________________________________________________
_______________________________________________________________
_______________________________________________________________
_______________________________________________________________
5. _______________________________________________________________
_______________________________________________________________
_______________________________________________________________
_______________________________________________________________

Q & A List

List 1: After series of discussion and exercises, kindly list down your questions or issues
in conjunction with the topics. Write the answers after clarification under the column
provided. This portion helps you in the review of concepts and essential knowledge.

Do you have any questions for clarification?


Questions/Issues Answers
1.
2.
3.
4.
5.

Keywords Index

Index: The keywords presented here will help you remember the concepts and essential
knowledge. Here are the few subject indices:

A C D
Annuity Costs Deferred annuity
Annuity Due Capital

O P
Ordinary Annuity Perpetuity

49
College of Engineering Education
2nd Floor, B&E Building
Matina Campus, Davao City
Telefax: (082) 296-1084
Phone No.: (082)300-5456/300-0647 Local 133

Big Picture in Focus:


ULO-2b. Understand and solve for the real-life economic situations involving
the concept of the uniform gradient amount.

Metalanguage
This section will be our basis in dealing with the terms we will encounter in this
section.

1. Gradient. This is the amount of the increase or decrease in a cash flow. It can also
be simply called as the amount of change.
2. Uniform Gradient Amount. This refers to the amount that repeats at the end of
each year, starting at the end of the second (2nd) year and stopping at the end of the
nth year.
Essential Knowledge
In ULO-2b, we will study a concept which is a bit similar to the annuity problems.
You will still deal with a number of equations which you need to comprehend and
familiarize. It is highly recommended to practice solving problems involving uniform
gradients.

1. Gradient Series. It is the series of cash flows where the amounts change every
period. It is categorized into two (2):

a. Arithmetic Gradient. It is the increase or decrease by a relatively constant


amount of each period. This increase or decrease may occur for maintenance
and repair expenses on a specific equipment or property.

Fig. 2b-1. Arithmetic Gradient Cash


Flow Diagram

*Fig. 2b-1 is equal to the sum of the


cash flows shown in Fig. 2b-2.
Fig. 2b-2. Elaborated CFD for
Arithmetic Gradient

50
College of Engineering Education
2nd Floor, B&E Building
Matina Campus, Davao City
Telefax: (082) 296-1084
Phone No.: (082)300-5456/300-0647 Local 133

In solving for the problems with uniform arithmetic gradient, we denote the
difference between two preceding amounts (increase per period) as G, which is also
known as uniform gradient amount. Fig2b-2 can be analyzed by the following formulas:

𝑃 = 𝑃𝐴 + 𝑃𝐺 (2b.1)

𝐴1 (1 + 𝑖)𝑛 − 1 (2b.2)
𝑃𝐴 = [ ]
𝑖 (1 + 𝑖)𝑛
𝐺 (1 + 𝑖)𝑛 − 1
𝑃𝐺 = [ − 𝑛] (1 + 𝑖)−𝑛 (2b.3)
𝑖 𝑖
1 𝑛
𝐴𝐺 = 𝐺 [ − ] (2b.4)
𝑖 (1 + 𝑖)𝑛 − 1

𝐴1 [(1 + 𝑖)𝑛 − 1] 𝐺 (1 + 𝑖)𝑛 − 1


𝐹 = 𝑃(1 + 𝑖)𝑛 = + [ − 𝑛] (2b.5)
𝑖 𝑖 𝑖
Where:
F = value or sum of money at some future time
P = present worth of money or value of money at present
A1 = cash flow in period 1
G = uniform gradient amount
i = interest rate per interest period
n = number of interest periods/number of equal payments

Example 2b.1: The XYZ Highway Department expects the cost of the maintenance for
a particular piece of heavy equipment to be ₱5,000 in the 1st year, ₱5,500 in the 2nd
year, and amounts increasing by ₱500 through year 10. At an interest rate of 10% per
year, the present worth of maintenance cost is what?

Given: Solution:
𝐴 = ₱5,000 To solve for the Present Worth use the formula:
𝐺 = ₱500 𝑃 = 𝑃𝐴 + 𝑃𝐺
𝑛 = 10
𝐴 (1+𝑖)𝑛 −1 𝐺 (1+𝑖)𝑛−1 −𝑛
𝑖 = 10% 𝑜𝑟 0.1 𝑃= [
𝑖 (1+𝑖)𝑛
]+
𝑖
[
𝑖
− 𝑛] (1 + 𝑖)
Now, substitute all the given values to the formula:
Required:
10
₱5000 (1+0.1)10 −1 ₱500 (1+0.1) −1
𝑃 =? 𝑃=
0.1
[
(1+0.1)10
]+
0.1
[
0.1
− 10] (1 + 0.1)−10

𝑃 = ₱30,722.84 + ₱11,445.67

𝑃 = ₱𝟒𝟐, 𝟏𝟔𝟖. 𝟓𝟏

51
College of Engineering Education
2nd Floor, B&E Building
Matina Campus, Davao City
Telefax: (082) 296-1084
Phone No.: (082)300-5456/300-0647 Local 133

Example 2b.2: The cash flow associated with a strip-mining operation is expected to
be ₱200,000 in year 1, ₱180,000 in year 2, and amounts decreasing by ₱20,000 per
year through 8. At an interest rate of 12% per year, determine the equivalent annual
cash flow.

Given: Solution:
𝐴1 = ₱200,000 To solve for the annual cash flow use the formula:
𝐺 = −₱20,000 𝐴 𝑇 = 𝐴1 + 𝐴𝐺
𝑛=8
𝑖 = 12% 𝑜𝑟 0.12 Now, use the formula for 𝐴𝐺 and substitute all the given values to the
above formula:
1 8
Required: 𝐴 𝑇 = ₱200,000 + (−₱20,000) [ − ]
0.12 (1+0.12)8 −1
𝐴 𝑇 =?
𝐴 𝑇 = ₱200,000 − ₱58,262.88

𝐴 𝑇 = ₱𝟏𝟒𝟏, 𝟕𝟑𝟕. 𝟏𝟐
b. Geometric Gradient Series. It is one of the two (2) types of gradient series
where deposits or payments increase or decrease by a fixed percentage, G.

𝐼𝑓 𝐺 ≠ 𝑖:
1+𝐺 𝑛
𝐴1 [1 − ( ) ]
1+𝑖 (2b.6)
𝑃=
𝑖−𝐺
𝐼𝑓 𝐺 = 𝑖:
𝑛
𝑃 = 𝐴1 [ ] (2b.7)
1+𝑖
Where:
P = present worth of money or value of money at present
A = cash flow in period 1
G = rate of change per period
i = effective interest rate period
n = number of interest periods/number of equal payments

Example 2b.3: A mechanical contractor is trying to calculate the present worth of


personnel salaries over the next five years. He has four employees whose combined
salaries thru the end of this year are ₱150,000. If he expects to give each employee a
raise of 5% each year, the present worth of his employees’ salaries at an interest rate
of 12% per year is what?
Given: Solution:
𝐴1 = ₱150,000
1+𝐺 𝑛 1 + 0.05 5
𝐺 = 0.05 𝐴1 [1 − ( ) ] ₱15000 [1 − ( ) ]
1+𝑖 1 + 0.12
𝑛=5 𝑃= =
𝑖 = 12% 𝑜𝑟 0.12 𝑖−𝐺 0.12 − 0.05
Required: 𝑃 =?
𝑃 = ₱𝟓𝟗𝟏, 𝟎𝟎𝟕. 𝟔
52
College of Engineering Education
2nd Floor, B&E Building
Matina Campus, Davao City
Telefax: (082) 296-1084
Phone No.: (082)300-5456/300-0647 Local 133

Example 2b.4: Suppose that the maintenance for a piece of equipment costs ₱300
end-of-year 1 (EOY1) and increases by 15% every year for 5 years. The value of money
is 9%. What is the equivalent present cost over the time horizon?

Given: Solution:
𝐴 = ₱300
𝐺 = 0.15 1+𝐺 𝑛 1 + 0.15 5
𝑛=5 𝐴 [1 − ( ) ] ₱300 [1 − (1 + 0.09) ]
1+𝑖
𝑖 = 9% 𝑜𝑟 0.9 𝑃= =
𝑖−𝐺 0.09 − 0.15
Required: 𝑃 =?
𝑃 = ₱𝟏𝟓𝟑𝟔. 𝟐𝟐

Self-Help: You can also refer to the sources below to help you and guide you
further understand the lesson

Park, Chan S. (2011), Contemporary Engineering Economics (5th Edition), New Jersey:
Pearson Education, Chapters 8, 9, 11

Blank, L T., (2012), Engineering Economy (7th Edition), New York: McGraw Hill,
Chapter 9, 10, 11, 12, 13, 16, 17

Sta. Maria, H., Engineering Economy (3th Edition), National Book Store

Panneerselvam, R., (2012), Engineering Economics (Eastern Economy Edition), New


Delhi: PHI Learning Private Limited

Lindeburg, M., (2014), Civil Engineering Reference Manual for the PE Exam (14th
Edition), California: Professional Publications, Inc.

Let’s Check
Activity 1: Now that you review the most essential principles in engineering economy.
Let us try to check your understanding by solving the following cost problems:

1. A loan was to be amortized by a group of four end-of-year payments forming an


ascending arithmetic progression. The initial payment was to be ₱5,000 and the
difference between successive payments was to be ₱400. But the loan was
renegotiated to provide for the equal payment rather than uniformly varying sums. If the
interest rate of the loan is 15%, what was the annual payment?

53
College of Engineering Education
2nd Floor, B&E Building
Matina Campus, Davao City
Telefax: (082) 296-1084
Phone No.: (082)300-5456/300-0647 Local 133

2. Find the equivalent annual payment of the following obligations at 20% interest.

End of
Payment
Year
1 ₱8,000
2 7,000
3 6,000
4 5,000

Let’s Analyze
Activity 1: Solve the following problems and box the final answer.

1. A contract has been signed to lease a building at ₱200,000 per year with an annual
increase of ₱1,500 for 8 years. Payments are to be made at the end of each year,
starting one year from now. The prevailing interest rate is 7%. What lump sum paid
today would be equivalent to the 8-year lease-payment plan?
2. Determine the present worth if ₱400 is paid in year 1 and amounts increasing by
₱30 per year through year 5 at an interest rate of 12% per year.
3. The maintenance of an old machine is ₱1,500 this year but is expected to increase
by ₱125 each year thereafter. What present sum of money would need to be set
aside now to pay the maintenance for an 8-year period? Assume 10% interest rate.
4. Use a geometric gradient formula to compute the present value, P, for the following
cash flows where i=15%:
266.20
242
220
200

0 1 2 3 4

In a Nutshell
Activity 1: Based from the definition of the most essential terms in the study of
curriculum and the learning exercises that you have done, please feel free to write your
arguments or lessons learned below. I have indicated my arguments or lessons learned.
1. The study of uniform gradient amounts is necessary for real-life situations such
as for maintenance and repair.
Your Turn

2. _______________________________________________________________
_______________________________________________________________

54
College of Engineering Education
2nd Floor, B&E Building
Matina Campus, Davao City
Telefax: (082) 296-1084
Phone No.: (082)300-5456/300-0647 Local 133

_______________________________________________________________
_______________________________________________________________
3. _______________________________________________________________
_______________________________________________________________
_______________________________________________________________
_______________________________________________________________
4. _______________________________________________________________
_______________________________________________________________
_______________________________________________________________
_______________________________________________________________
5. _______________________________________________________________
_______________________________________________________________
_______________________________________________________________
_______________________________________________________________

Q & A List

List 1: After series of discussion and exercises, kindly list down your questions or issues
in conjunction with the topics. Write the answers after clarification under the column
provided. This portion helps you in the review of concepts and essential knowledge.

Do you have any questions for clarification?


Questions/Issues Answers
1.
2.
3.
4.
5.

Keywords Index

Index: The keywords presented here will help you remember the concepts and essential
knowledge. Here are the few subject indices:

A G M R
Annuity Gradient Maintenance Repair
Arithmetic Gradient

U
Uniform Gradient

55
College of Engineering Education
2nd Floor, B&E Building
Matina Campus, Davao City
Telefax: (082) 296-1084
Phone No.: (082)300-5456/300-0647 Local 133

Big Picture
Week 6-7: Unit Learning Outcomes-Unit 3 (ULO-3): At the end of the unit, you are
expected to:
c. Apply several depreciation methods in real-life applications such as straight-
line method, sum of year’s digit method, declining balance method, double
declining balance method and sinking fund method; and
d. Solve for the capitalized and annual cost of an asset over the course of its
useful life for the cost comparison of different alternatives.

Big Picture in Focus:


ULO-3a. Apply several depreciation methods in real-life applications such as
straight-line method, sum of year’s digit method, declining balance method,
double declining balance method and sinking fund method.

Metalanguage
The most essential terms below are defined for you to have a better
understanding of this section in the course.

1. Depreciation. It is the decrease in the value of a physical property with the passage
of time.
1.1 It represents how much of an asset’s value has been used up.
1.2 It is an accounting method of allocating the cost of a tangible or physical asset
over its useful life or life expectancy.
1.3 It is the monetary value of an asset that decreases over time due to use, wear
and tear, or obsolescence.
2. Value. It is the present worth of all future profits that are to be received through the
ownership of a particular property.
1.1 It is the amount of money that can be received from something.
3. Market Value. It is the amount which a willing buyer will pay to a willing seller for the
property where each has equal advantage and is under no compulsion to buy or sell.
3.1 It is a generic term that represents the price an asset would get in a marketplace.
4. Utility Value. It is what the property is worth to the owner as an operating unit.
4.1 It is a subjective assessment of the expected return on an investment at a given
risk.
5. Fair Value. It is the value which is usually determined by a disinterested third-party
in order to establish a price that is fair to both seller and buyer.
6. Book Value. It can sometimes be called as depreciated book value.
6.1 It is the worth of a property that is shown on the accounting records of an
enterprise.
6.2 It literally means the value of a business according to its books (accounts) that
is reflected through its financial statements.

56
College of Engineering Education
2nd Floor, B&E Building
Matina Campus, Davao City
Telefax: (082) 296-1084
Phone No.: (082)300-5456/300-0647 Local 133

7. Salvage Value. It can also be referred to as the resale value.


7.1 It is the price that can be obtained from the sale of a property after it has been
used.
8. Scrap Value. It is the amount the property would sell if disposed off as a junk.
8.1 It is the worth of a physical asset’s individual components when the asset itself
is deemed no longer usable.
9. Physical Life of a Property. It is the length of time during which it is capable of
performing the function for which it was designed and manufactured.
10. Economic Life. It is the length of time during which the property may be operated
at a profit.
Essential Knowledge
To perform the aforesaid big picture (unit learning outcomes) for the week 4-5 of
this course, you need to fully understand the following essential knowledge that will be
laid down on the following pages. Please note that you are not limited to refer to these
resources exclusively. Thus, you are expected to utilize other books, research articles,
and other resources that are available in the university’s library.
1. Types of Depreciation. There are various types of depreciation, and these explain
why depreciation occurs. These are the following:
a. Normal depreciation
i. Physical- is due to the lessening of physical ability of a property to
produce results; common causes are wear and deterioration
ii. Functional- is due to the lessening in the demand for the function which
the property was designed to render; common causes are
inadequacy, changes in styles, population center shift,
a saturation of markets or more efficient machines are
produced
b. Depreciation due to changes in price levels
- is almost impossible to predict and therefore is not considered in
economy studies
c. Depletion
- it refers to the decrease in the value of a property due to the gradual
extraction of its contents
2. Depreciation Methods. Several methods will be discussed on the succeeding
pages. Take note of the formula below since this can be used in any method.
Where:
𝑩𝑽𝒓 = 𝑭𝑪 − 𝑫𝒓 BVr = book value of a property at any time r
Dr = total depreciation of a property at any time r
𝑫𝒏 = 𝑭𝑪 − 𝑺𝑽 Dn = total depreciation at the end of its useful life n
FC = first cost
SV = salvage or scrap value

57
College of Engineering Education
2nd Floor, B&E Building
Matina Campus, Davao City
Telefax: (082) 296-1084
Phone No.: (082)300-5456/300-0647 Local 133

a. Unit Production Method. This method is applied where the value is more
closely related to the number of units it produces. The estimated total
production of the asset is the criteria for providing depreciation. Thus, in the
years when the asset is heavily used, the amount of depreciation is high.

Study the given graph and familiarize the symbols used in this depreciation
method.

Example 3a.1: A Xerox machine costing ₱75,000 with a salvage value of ₱10,000 at
the end of its life can print 3,000,000 sheets in its entire life. If it has already printed
500,000 sheets, determine the total depreciation of the Xerox machine at that time.

Given: Solution:
𝐹𝐶 = ₱75,000 Solve for 𝑫𝒏 , first, using 𝐷𝑛 = 𝐹𝐶 − 𝑆𝑉
𝑆𝑉 = ₱10,000
𝑛 = 3,000,000 𝐷𝑛 = ₱75,000 − ₱10,000 = ₱65,000
𝑟 = 500,000 Now, find 𝑫𝒓 by:

Required: 𝐷𝑟 =? (𝑟)(𝐷𝑛 )
𝐷𝑟 = 𝑛

(500,000)(₱65,000)
𝐷𝑟 = 3,000,000

𝐷𝑟 = ₱𝟏𝟎, 𝟖𝟑𝟑. 𝟑𝟑

58
College of Engineering Education
2nd Floor, B&E Building
Matina Campus, Davao City
Telefax: (082) 296-1084
Phone No.: (082)300-5456/300-0647 Local 133

b. Working Hours Method. It is calculated based on the usage of machinery,


which is usually directly proportional to the number of machine-hours. It is
widely used for assets like plant, machinery, vehicles, etc.

Example 3a.2: A machine costing ₱3,000,000 has a salvage value of ₱0.1M at the end
of its useful life, 20 years. It can be used for 5,000 hours in its entire life. Find the
accumulated depreciation after using it for 1,000 hours.

Given: Solution:
𝐹𝐶 = ₱3,000,000 Solve for 𝑫𝒏 , first, using 𝐷𝑛 = 𝐹𝐶 − 𝑆𝑉
𝑆𝑉 = ₱0.1 M
𝑛 = 5,000 𝐷𝑛 = ₱3M − ₱0.1M = ₱2.9M
𝑟 = 1,000 Now, find 𝑫𝒓 by:

Required: 𝐷𝑟 =? (𝑟)(𝐷𝑛 )
𝐷𝑟 =
𝑛

(1,000)(₱2.9M)
𝐷𝑟 = 5,000

𝐷𝑟 = ₱𝟓𝟖𝟎, 𝟎𝟎𝟎

59
College of Engineering Education
2nd Floor, B&E Building
Matina Campus, Davao City
Telefax: (082) 296-1084
Phone No.: (082)300-5456/300-0647 Local 133

c. Straight-Line Method. It is a common method of depreciation where the


value of a fixed asset is reduced gradually over its useful life. With this
method, the value of an asset is reduced uniformly over each period until it
reaches its salvage value.

𝑑1 = 𝑑2 = ⋯ = 𝑑𝑟 = 𝑑𝑛 = 𝑑
𝐹𝐶 − 𝑆𝑉
𝑑=
𝑛
𝐷𝑟 = 𝑟𝑑

𝐷𝑛 = 𝑛𝑑

Example 3a.3: A tax- and duty-free importation of a 30hp sand mill (for paint
manufacturing) cost ₱360,000. CIP Manila Bank charges, arrester, and brokerage cost
₱5,000. Foundation and installation cost were ₱25,000. Other incidental expenses
amounted to ₱20,000. Salvage value of the mill is estimated to be ₱60,000 after 20
years. Determine the appraisal value of the mill using straight line method of
depreciation at the end of 10 years.

Given: Solution:
𝐹𝐶 = ₱360,000 + ₱5,000 Solve for total 𝐹𝐶, first:
+ ₱25,000 + ₱20,000 𝐹𝐶 = ₱360,000 + ₱5,000 + ₱25,000 + ₱20,000
𝑆𝑉 = ₱60,000 𝐹𝐶 = ₱410,000
𝑛 = 20
𝑟 = 10 Next, find 𝑑 by:
𝐹𝐶−𝑆𝑉 ₱410,000−₱60,000
𝑑= = = ₱17,500
Required: 𝐵𝑉10 =? 𝑛 20
Solve for 𝐷10 :
𝐷10 = 𝑟𝑑 = 10(₱17,500) = ₱175,000
Lastly, find the 𝐵𝑉10 :
𝐵𝑉10 = 𝐹𝐶 − 𝐷10
𝐵𝑉10 = ₱410,000 − ₱175,000
𝐵𝑉10 = ₱𝟐𝟑𝟓, 𝟎𝟎𝟎
60
College of Engineering Education
2nd Floor, B&E Building
Matina Campus, Davao City
Telefax: (082) 296-1084
Phone No.: (082)300-5456/300-0647 Local 133

d. Sinking Fund Method. It is a technique for depreciating an asset while


generating enough money to replace it at the end of its useful life. While
depreciation charges are experienced reflecting the asset’s falling value, a
matching amount of cash is invested. These funds sit in a sinking fund
account and generate interest.
𝒅𝟏 = 𝒅𝟐 =. . . = 𝒅𝒓 = 𝒅𝒏 = 𝒅
(𝑭𝑪 − 𝑺𝑽)𝒊
𝒅=
(𝟏 + 𝒊)𝒏 − 𝟏
𝒅
𝑫𝒓 = [(𝟏 + 𝒊)𝒓 − 𝟏]
𝒊
𝒅
𝑫𝒏 = [(𝟏 + 𝒊)𝒏 − 𝟏]
𝒊
Where:
d = depreciation at any year
Dr = total depreciation of a property at any time r
Dn = total depreciation at the end of its useful life
n = useful life in years
FC = first cost
SV = salvage or scrap value
i = interest rate

Example 3a.4: A contractor imported a bulldozer for his manufacturer paying ₱250,000.
Freight and insurance amounted to ₱18,000, while broker’s fees and arrester services,
₱8,500. Taxes, permits, and others is ₱25,000. If the contractor estimates the life of the
bulldozer to be 10 years with a salvage value of ₱20, 000, determine the book value at
the end of 6 years using sinking fund method at 8% interest.
Given: Solution:
𝐹𝐶 = ₱250,000 + ₱18,000
Solve for total 𝐹𝐶, first:
+ ₱8,500 + ₱25,000
𝑆𝑉 = ₱20,000 𝐹𝐶 = ₱250,000 + ₱18,000 + ₱8,500 + ₱25,000
𝑛 = 10 𝐹𝐶 = ₱301,500
𝑟=6 Next, find 𝑑 by:
𝑖 = 8% (𝐹𝐶−𝑆𝑉)𝑖 (₱301,500−₱20,000)(0.08)
𝑑 = (1+𝑖)𝑛 −1 = = ₱19,431.80
(1+0.08)10 −1
Required: Solve for 𝐷6 :
𝐵𝑉6 =? 𝑑
𝐷6 = 𝑖 [(1 + 𝑖)𝑟 − 1]
₱19431.80
= 0.08 [(1 + 0.08)6 − 1]
= ₱142550.31
Lastly, find the 𝐵𝑉6 :
𝐵𝑉6 = 𝐹𝐶 − 𝐷6 = ₱301,500 − ₱142,550.31
𝐵𝑉6 = ₱𝟏𝟓𝟖, 𝟗𝟒𝟗. 𝟔𝟗
61
College of Engineering Education
2nd Floor, B&E Building
Matina Campus, Davao City
Telefax: (082) 296-1084
Phone No.: (082)300-5456/300-0647 Local 133

e. Declining Balance Method. This method is also called the Matheson’s


Method. It is an accelerated depreciation system of recording larger
depreciation expenses during the earlier years of an asset’s useful life while
recording smaller depreciation expenses during the asset’s later years. This
technique represents the opposite of the straight-line depreciation.

𝒅𝒓 = 𝑭𝑪(𝟏 − 𝒌)𝒓−𝟏𝒌
𝑩𝑽𝒓 = 𝑭𝑪(𝟏 − 𝒌)𝒓 Note: This method is not
𝒏 applicable if there is no
𝑺𝑽 = 𝑭𝑪(𝟏 − 𝒌) Salvage Value.

𝒏 𝑺𝑽
𝒌=𝟏− √
𝑭𝑪

𝑫𝒓 = 𝑭𝑪 − 𝑩𝑽𝒓
Where:
dr = depreciation at any time r
BVr = book value of a property at any time r
Dr= total depreciation of a property at any time r
n = useful life in years
FC = first cost
SV = salvage or scrap value
k = rate of depreciation

Example 3a.5: An industrial plant bought a generator set for ₱90,000. Other expenses
including installation amounted to ₱10,000. The generator set is to have a life of 17
years with a salvage value at the end of life at ₱5,000. Determine the depreciation
charge during the 13th year and the book value at the end of 13 years by the declining
balance method.
Given: Solution:
𝐹𝐶 = ₱90,000 + ₱10,000
Solve for total 𝐹𝐶, first:
𝑆𝑉 = ₱5,000
𝑛 = 17 𝐹𝐶 = ₱90,000 + ₱10,000 = ₱100,000
𝑟 = 13 Next, find 𝑘 by:
𝑛 𝑆𝑉 17 ₱5000
Required: 𝑘 = 1− √ =1− √ = 0.1616
𝐹𝐶 ₱100000
𝑑13 =?
𝐵𝑉13 =? Solve for 𝑑13 :
𝑑13 = 𝐹𝐶(1 − 𝑘)𝑟−1 𝑘
= ₱100000(1 − 0.1616)13−1 (0.1616)
= ₱𝟏𝟗𝟒𝟗. 𝟐𝟎
Lastly, find the 𝐵𝑉6 :
𝐵𝑉13 = 𝐹𝐶(1 − 𝑘)𝑟
= ₱100000(1 − 0.1616)13
= ₱𝟏𝟎, 𝟏𝟏𝟐. 𝟔𝟗

62
College of Engineering Education
2nd Floor, B&E Building
Matina Campus, Davao City
Telefax: (082) 296-1084
Phone No.: (082)300-5456/300-0647 Local 133

f. Double Declining Balance Method. This technique can also be referred to


as the reducing balance method. It is a form of accelerated depreciation
which means that compared to the straight-line method, the depreciation
expense will be faster in early years of the asset’s life but slower in the later
years.

𝟐 𝒓−𝟏 𝟐
𝒅𝒓 = 𝑭𝑪 − )
(𝟏
𝒏 𝒏
𝟐 𝒓
𝑩𝑽𝒓 = 𝑭𝑪 (𝟏 − )
𝒏
𝑫𝒓 = 𝑭𝑪 − 𝑩𝑽𝒓
Where:
dr = depreciation at any time r
BVr = book value of a property at any time r
Dr= total depreciation of a property at any time r
n = useful life in years
FC = first cost

Example 3a.6: Vectors Corporation owns earth moving equipment that cost ₱90,000.
After 8 years, it will have estimated salvage value of ₱18,000. Compute the depreciation
charge using double declining balance method at the end of 2nd year and the book value
at the end of 5th year.

Given: Solution:
𝐹𝐶 = ₱90,000 Solve for total 𝑑2 :
𝑆𝑉 = ₱18,000
2 2−1 2
𝑛= 8 𝑑2 = 𝐹𝐶 (1 − )
𝑛 𝑛
𝑟 = 2;5 2 2−1 2
= ₱90000 (1 − )
8 8
Required: = ₱𝟏𝟔𝟖𝟕𝟓
𝑑2 =? Next, find 𝐵𝑉5 by:
𝐵𝑉5 =?
2 𝑟
𝐵𝑉5 = 𝐹𝐶 (1 − )
𝑛
2 5
= ₱90000 (1 − )
8
= ₱𝟐𝟏𝟑𝟓𝟕. 𝟒𝟐

63
College of Engineering Education
2nd Floor, B&E Building
Matina Campus, Davao City
Telefax: (082) 296-1084
Phone No.: (082)300-5456/300-0647 Local 133

g. Sum of the Year’s Digit Method. This method is also an accelerated


depreciation. This method is more appropriate than the most commonly used
straight-line depreciation if an asset depreciates more quickly or has greater
production capacity in its earlier years than it does as it ages.

(𝐹𝐶 − 𝑆𝑉)
𝑑𝑚 = (𝑟𝑒𝑣𝑒𝑟𝑠𝑒 𝑑𝑖𝑔𝑖𝑡)
𝑠𝑢𝑚 𝑜𝑓 𝑡ℎ𝑒 𝑑𝑖𝑔𝑖𝑡𝑠
(𝐹𝐶 − 𝑆𝑉)
𝐷𝑚 = (𝑠𝑢𝑚 𝑜𝑓 𝑟𝑒𝑣𝑒𝑟𝑠𝑒 𝑑𝑖𝑔𝑖𝑡𝑠)
𝑠𝑢𝑚 𝑜𝑓 𝑡ℎ𝑒 𝑑𝑖𝑔𝑖𝑡𝑠
𝑛
𝑠𝑢𝑚 𝑜𝑓 𝑡ℎ𝑒 𝑑𝑖𝑔𝑖𝑡𝑠 = (𝑛 + 1)
2
𝑟𝑒𝑣𝑒𝑟𝑠𝑒 𝑑𝑖𝑔𝑖𝑡 = 𝑛 − 𝑟 + 1
𝑟
𝑠𝑢𝑚 𝑜𝑓 𝑟𝑒𝑣𝑒𝑟𝑠𝑒 𝑑𝑖𝑔𝑖𝑡𝑠 = (2𝑛 − 𝑟 + 1)
2

Where:
dr = depreciation at any time r
Dr = total depreciation of a property at any time r
n = useful life in years
FC = first cost
SV = salvage or scrap value

Example 3a.7: A machine is purchased for ₱100,000 with an estimated useful life of
10years, after which it will be sold for ₱20,000. Find the book value at the end of the
third year using sum of year’s digit method (SYDM).

Given: Solution:
𝐹𝐶 = ₱100,000 Solve for total 𝑠𝑢𝑚 𝑜𝑓 𝑑𝑖𝑔𝑖𝑡𝑠:
𝑆𝑉 = ₱20,000 𝑠𝑢𝑚 𝑜𝑓 𝑑𝑖𝑔𝑖𝑡𝑠 = (𝑛 + 1)
𝑛
𝑛 = 10 10
2

𝑟=3 = (10 + 1) = 55
2
Next, solve for 𝑠𝑢𝑚 𝑜𝑓 𝑟𝑒𝑣𝑒𝑟𝑠𝑒 𝑑𝑖𝑔𝑖𝑡𝑠 by:
Required: 𝑟
𝐵𝑉3 =? 𝑠𝑢𝑚 𝑜𝑓 𝑟𝑒𝑣𝑒𝑟𝑠𝑒 𝑑𝑖𝑔𝑖𝑡𝑠 = (2𝑛 − 𝑟 + 1)
2
3
= (2(10) − 3 + 1) = 27
2
Find 𝐷𝑟 :
𝐹𝐶−𝑆𝑉
𝐷3 = (𝑠𝑢𝑚 𝑜𝑓 𝑟𝑒𝑣𝑒𝑟𝑠𝑒 𝑑𝑖𝑔𝑖𝑡𝑠)
𝑠𝑢𝑚 𝑜𝑓 𝑑𝑖𝑔𝑖𝑡𝑠

₱100000−₱20000
= (27) = ₱39272.73
55

Lastly, solve for:


𝐵𝑉3 = 𝐹𝐶 − 𝐷3 = ₱100000 − ₱39272.73 = ₱𝟔𝟎𝟕𝟐𝟕. 𝟐𝟕

64
College of Engineering Education
2nd Floor, B&E Building
Matina Campus, Davao City
Telefax: (082) 296-1084
Phone No.: (082)300-5456/300-0647 Local 133

Self-Help: You can also refer to the sources below to help and guide you
to further understand the lesson

Park, Chan S. (2011), Contemporary Engineering Economics (5th Edition), New Jersey:
Pearson Education, Chapters 8, 9, 11

Blank, L T., (2012), Engineering Economy (7th Edition), New York: McGraw Hill,
Chapter 9, 10, 11, 12, 13, 16, 17

Sta. Maria, H., Engineering Economy (3th Edition), National Book Store

Panneerselvam, R., (2012), Engineering Economics (Eastern Economy Edition), New


Delhi: PHI Learning Private Limited

Lindeburg, M., (2014), Civil Engineering Reference Manual for the PE Exam (14th
Edition), California: Professional Publications, Inc.

Let’s Check

Activity 1: At this point, it is assumed that have gained enough knowledge to answer
the practice problems below. Copy and answer the following word problems and show
a clear solution for each.

1. A certain equipment costs ₱7,000 has an economic life of “n” years and a salvage
value of ₱350 at the end of “n” years. If the book value at the end of 4 years is equal
to ₱2197.22, compute the economic life of the equipment using SYDM.

2. A firm bought an equipment for ₱56,000. Other expenses including installation


amounted to ₱4,000. The equipment is expected to have a life of 16 years with a
salvage value of 10% of the original cost. Determine the book value at the end of 12
years by sinking fund method at 12% interest.

3. A broadcasting corporation purchased an equipment for ₱53,000 and paid ₱1,500


for freight and delivery charges to the job site. The equipment has a normal life of
10 years with a trade-in-value of ₱5,000 against the purchase of a new equipment
at the end of its life. Determine the annual depreciation cost by the straight-line
method.

4. A plant bought a calciner for ₱220,000 and used it for 10 years, the life span of the
equipment. What is the book value of the calciner after 5 years of use? Assume a
scrap value of ₱22,000 for declining balance method.

65
College of Engineering Education
2nd Floor, B&E Building
Matina Campus, Davao City
Telefax: (082) 296-1084
Phone No.: (082)300-5456/300-0647 Local 133

5. An equipment cost P480,000 and has a salvage value of 10% of its cost at the end
of its life of 36,000 operating hours in a period of 5 years. If, at the end of the second
year, it was used for 15,000 hours, find the depreciation at the end of the second
year.

6. A machine was purchased at an original cost of P400,000 with a salvage value of


P20,000. Life of this machine is expected to last for 6 years. It was used for 4000
hrs. in the first year, 6000 hrs. in the second year, and 8000 hrs. in the third year.
The machine is expected to last for 38,000 hrs. in a period of 6 yrs. Find the
depreciation at the end of the second year.

7. A lathe machine has an estimated salvage value of P10,000 at the end of its useful
life of 8 yrs. Compute the first cost of the machine if the total depreciation at the end
of 4th year using double declining balance method is P957,031.25.

Let’s Analyze

Activity 1: Solve the following problems and state all the givens and its requirement.

1. A radio service panel truck initially cost ₱56,000. Its resale value at the end of the 5th
year of the useful life is estimated at ₱15,000. By means of the declining balance
method, determine the depreciation charge for the 2 nd year.

2. A machine costs ₱7,000, lasts 8 years and has a salvage value at the end of its life
of ₱350. Determine the depreciation charge during the 4th year and the book value
at the end of 4 years by the sinking fund method with interest of 12%.

3. A motor cost ₱100,000 has an estimated life of 10 years and can be sold at ₱20,000
at the end of 10 years. Calculate the book value at the end of 8 years using sum of
year’s digit method (SYDM).

4. The original cost of a certain machine is ₱150,000 has an economic life of 8 years
with a salvage value of ₱9000 at that time. If the depreciation of the first year is equal
to ₱44475, what method is used in the calculation of depreciation?

5. Power to a remote transmitting station is provided by a Diesel-electric generator unit.


The original cost of the unit is ₱65,000. It costs ₱2,000 to ship the unit to the job
site. An additional cost of ₱3,000 was incurred for installation. Determine the annual
depreciation cost by the straight-line method, if the unit has an expected life of 10
years. The salvage value of the unit at the end of its life was estimated at ₱5,000.

6. ABC Corporation makes its policy that for every new equipment purchased, the
annual depreciation cost should not exceed 20% of the first cost at any time without
salvage values. Determine the length of service if the depreciation used is SYDM.

66
College of Engineering Education
2nd Floor, B&E Building
Matina Campus, Davao City
Telefax: (082) 296-1084
Phone No.: (082)300-5456/300-0647 Local 133

7. A certain machine cost ₱40,000 and has a life of 4 years and a salvage value of
₱5000. The production output of this machine in units per year is 1000 units for the
first year, 2000 units for the second year, 3000 units for the third year, and 4000
units for the fourth year. If the units produced are in uniform quality, what is
the depreciation charge at the end of the fourth year.

8. An earth moving equipment that cost P90,000 will have an estimated salvage value
of P18,000 at the end of 8 years. Using double-declining balance method, compute
the book value and the total depreciation at the end of the 5th year.

9. A machine has a first of P80,000 and a salvage of P2,000 at the end of its life of 10
years. Find the book value at the end of the 6th year using straight line method of
depreciation.

10. A machine having a certain first cost has a life of 10 years and a salvage value of
6.633% of the first cost at the end of 10 years. If it has a book value of P58,914 at
the end of the 6thyear, how much is the first cost of the machine if the constant
percentage of declining value is used in the computation for its depreciation. (Use
Matheson’s Method)

In a Nutshell

Activity 1: Based from the definition of the most essential terms in the study of
curriculum and the learning exercises that you have done, please feel free to write your
arguments or lessons learned below. I have indicated my lesson learned.

1. Straight line method is mostly used in equipment estimates.


Your Turn

2. _______________________________________________________________
_______________________________________________________________
_______________________________________________________________
_______________________________________________________________
_______________________________________________________________
3. _______________________________________________________________
_______________________________________________________________
_______________________________________________________________
_______________________________________________________________
_______________________________________________________________
4. _______________________________________________________________
_______________________________________________________________
_______________________________________________________________

67
College of Engineering Education
2nd Floor, B&E Building
Matina Campus, Davao City
Telefax: (082) 296-1084
Phone No.: (082)300-5456/300-0647 Local 133

_______________________________________________________________
_______________________________________________________________
5. _______________________________________________________________
_______________________________________________________________
_______________________________________________________________
_______________________________________________________________
_______________________________________________________________

Q & A List
List 1: After series of discussion and exercises, kindly list down your questions or issues
in conjunction with the topics. Write the answers after clarification under the column
provided. This portion helps you in the review of concepts and essential knowledge.

Do you have any questions for clarification?


Questions/Issues Answers
1.
2.
3.
4.
5.

Keywords Index
Index: The keywords presented here will help you remember the concepts and essential
knowledge. Here are the few subject indices.

C D F M
Constant Depreciation Fixed value Matheson’s
percentage of Double declining formula
depreciation balance method
Declining balance
method
S U W
Sinking fund Unit production Working hours
method method method
Sum of years digit
method
Scrap value

68
College of Engineering Education
2nd Floor, B&E Building
Matina Campus, Davao City
Telefax: (082) 296-1084
Phone No.: (082)300-5456/300-0647 Local 133

Big Picture in Focus:


ULO-3b. Solve for the capitalized and annual cost of an asset over the course
of its useful life for the cost comparison of different alternatives

Metalanguage
This section will serve as your word bank where the most essential terms relevant
to the introduction of engineering economy and ULO-3b will be operationally defined to
establish a common frame of reference. Please refer to these definitions in case you will
find it difficult to understand.
1. Capitalized Cost. It is one of the most important applications of perpetuity. It is an
expense that is added to the cost basis of a fixed asset on a company’s balance
sheet.
1.1 It refers to the present worth of cash flows which go on for an infinite period of
time. It is one method used in comparing alternatives.
2. Annual Cost. It is the cost per year of owning and operating an asset over its entire
lifespan.
2.1 It includes charges for depreciation, interest on the money invested, repairs for
normal use, property taxes, and insurance.
3. Operations and Maintenance. These are expenses associated with the
maintenance and administration of a business on a day-to-day basis.
4. Salvage Value. It is the amount for which the asset can be sold at the end of its
useful life.
4.1 It is the estimated resale value of an asset at the end of its useful life.
5. Replacement Cost. It is an amount that a company pays to replace an essential
asset that is priced at the same or equal value.
a. The cost to replace the asset can change, depending on the market value of the
asset and how much it costs to get the asset up and running, once purchased.

Essential Knowledge
To perform the aforesaid big picture (unit learning outcomes) for the unit 3 of this
course, you need to fully understand the following essential knowledge that will be laid
down on the following pages. Please note that you are not limited to refer to these
resources exclusively. Thus, you are expected to utilize other books, research articles,
and other resources that are available in the university’s library.

69
College of Engineering Education
2nd Floor, B&E Building
Matina Campus, Davao City
Telefax: (082) 296-1084
Phone No.: (082)300-5456/300-0647 Local 133

1. Capitalized Cost. It is the sum of the first cost and the present worth of all costs of
replacement, operation and maintenance for a long time or, even, forever. The
equation for getting the capitalized cost is:

𝑂𝑀 𝑅𝐶 − 𝑆𝑉
𝐾 = 𝐹𝐶 + +
𝑖 (1 + 𝑖)𝑛 − 1

*If RC is not specified, use RC=FC


Where:
𝑲 = 𝐶𝑎𝑝𝑖𝑡𝑎𝑙𝑖𝑧𝑒𝑑 𝐶𝑜𝑠𝑡
𝑭𝑪 = 𝐹𝑖𝑟𝑠𝑡 𝐶𝑜𝑠𝑡
𝑶𝑴 = 𝑂𝑝𝑒𝑟𝑎𝑡𝑖𝑜𝑛 𝑎𝑛𝑑 𝑀𝑎𝑖𝑛𝑡𝑒𝑛𝑎𝑛𝑐𝑒 𝐶𝑜𝑠𝑡
𝑹𝑪 = 𝑅𝑒𝑝𝑙𝑎𝑐𝑒𝑚𝑒𝑛𝑡 𝐶𝑜𝑠𝑡
𝑺𝑽 = 𝑆𝑎𝑙𝑣𝑎𝑔𝑒 𝑉𝑎𝑙𝑢𝑒
𝒊 = 𝑒𝑓𝑓𝑒𝑐𝑡𝑖𝑣𝑒 𝑟𝑎𝑡𝑒 𝑜𝑓 𝑖𝑛𝑡𝑒𝑟𝑒𝑠𝑡
𝒏 = 𝑛𝑢𝑚𝑏𝑒𝑟 𝑜𝑓 𝑦𝑒𝑎𝑟𝑠

Example 3b.1: Determine the capitalized cost at 15% interest of a structure with an
initial cost of ₱200,000 and annual operating and maintenance costs of ₱40,000.

Given: Solution:
𝐹𝐶 = ₱200,000 The problem did not mention about the RC, SV, and n. Therefore,
𝑂𝑀 = ₱40,000 we can assume that the 3rd term of the formula in finding for K is
𝑖 = 15% zero (0).
𝑂𝑀 𝑅𝐶−𝑆𝑉 Zero (0)
𝐾 = 𝐹𝐶 + + (1+𝑖)𝑛 −1
Required: 𝑖
𝐾 =? 𝑂𝑀
𝐾 = 𝐹𝐶 + +0
𝑖
₱40,000
= ₱200,000 +
0.15

= ₱𝟒𝟔𝟔, 𝟔𝟔𝟔. 𝟔𝟕

Example 3b.2: Find the capitalized cost of a bridge whose cost is ₱250M with a useful
life of 20 years at 6% interest and it must be partially rebuilt at a cost of ₱100M at the
end of each 20 years.
Given: Solution:
𝐹𝐶 = ₱250M The problem doesn’t have OM. Therefore, we can assume that
𝑅𝐶 − 𝑆𝑉 = ₱100M the 2nd term of the formula in finding for K is zero (0).
𝑖 = 6% 𝑂𝑀 𝑅𝐶−𝑆𝑉
𝐾 = 𝐹𝐶 + + (1+𝑖)𝑛 −1
𝑛 = 20 𝑖
Zero (0)
𝑅𝐶−𝑆𝑉
𝐾 = 𝐹𝐶 + 0 + (1+𝑖)𝑛 −1
Required: ₱100𝑀
= ₱250M +
𝐾 =? (1.06)20 −1

= ₱𝟐𝟗𝟓. 𝟑𝐌

70
College of Engineering Education
2nd Floor, B&E Building
Matina Campus, Davao City
Telefax: (082) 296-1084
Phone No.: (082)300-5456/300-0647 Local 133

2. Annual Cost. This allows comparison or joint consideration of items having different
lengths of life. For investments of buildings and equipment, which are expected to
last several years, it is reasonable to place their cost on an annual basis over several
years rather than a single year.

Calculation of annual cost is crude and can be improved using time value of
money to convert price and salvage to an annual cost, which includes automatically
both depreciation and interest.

If two (2) or more different articles are available for the same purpose, they are
equally economical if the corresponding present worth, annual cost or capitalized cost
are the same.

Example 3b.3: A certain equipment costs ₱150,000, lasts for 6 years, and has a
salvage value of ₱30,000. How much could an investor afford to pay for another
machine for the same purpose, whose life is 10 years and salvage value is ₱40,000, if
money is worth 5%?
Given: Solution:
𝐹𝐶1 = ₱150,000 K1 must be equal to K2 to find FC2
𝑅𝐶 = 𝐹𝐶
𝑆𝑉1 = ₱30,000 For the first machine:
𝑖 = 5% 𝑂𝑀 𝑅𝐶−𝑆𝑉1
𝐾1 = 𝐹𝐶1 + + (1+𝑖)𝑛 −1
𝑛1 = 6 𝑖
₱150,000−₱30,000
𝑆𝑉2 = ₱40,000 𝐾1 = ₱150,000 + 0 + (1+0.05)6−1
𝑛2 = 10
𝐾1 = ₱502,841.92
Required: For the other machine:
𝐹𝐶2 =? 𝑂𝑀 𝑅𝐶−𝑆𝑉2
𝐾1 = 𝐹𝐶2 + +
𝑖 (1+𝑖)𝑛 −1
𝐹𝐶2 −₱40,000
₱502,841.92 = 𝐹𝐶2 + 0 +
(1+0.05)10 −1
𝐹𝐶2 = ₱𝟐𝟏𝟖, 𝟔𝟗𝟔. 𝟒𝟏

Self-Help: You can also refer to the sources below to help and guide you
to further understand the lesson

Park, Chan S. (2011), Contemporary Engineering Economics (5th Edition), New Jersey:
Pearson Education, Chapters 8, 9, 11
Blank, L T., (2012), Engineering Economy (7th Edition), New York: McGraw Hill,
Chapter 9, 10, 11, 12, 13, 16, 17
Sta. Maria, H., Engineering Economy (3th Edition), National Book Store

71
College of Engineering Education
2nd Floor, B&E Building
Matina Campus, Davao City
Telefax: (082) 296-1084
Phone No.: (082)300-5456/300-0647 Local 133

Panneerselvam, R., (2012), Engineering Economics (Eastern Economy Edition), New


Delhi: PHI Learning Private Limited

Lindeburg, M., (2014), Civil Engineering Reference Manual for the PE Exam (14th
Edition), California: Professional Publications, Inc.

Let’s Check

Activity 1: Copy and answer the following word problems and show a clear solution for
each.

1. A dam will have a first cost of ₱5,000,000, an annual maintenance cost of ₱25,000
and minor reconstruction costs of ₱100,000 every five years. At an interest rate of
8% per year, determine the capitalized cost of the dam.

2. Determine the capitalized cost of an equipment costing ₱2M with an annual


maintenance of ₱200,000 if money is worth 20% per annum.

3. The first cost of a certain piece of equipment is ₱50,000. It will have an annual
operating cost of ₱20,000 and ₱5,000 salvage value after its 5-year life. At an interest
rate of 10% per year, calculate for the capitalized cost of the equipment.

Let’s Analyze

Activity 1: Solve the following problems and state all the givens and its requirement.

1. What is the capitalized cost of a public works project that will cost ₱25M now and
will require ₱2M in maintenance annually? The effective annual interest rate is 12%.

2. A municipality plans a pipeline to transport water from a distant watershed area to a


city. The pipeline will cost ₱8M and will have an expected life of 70 years. The water
line needs to be kept in service indefinitely. We estimate we need ₱8M every 70
years. Compute the capitalized cost where interest is 7% yearly.

3. A company uses a type of truck which costs ₱2M, with life of 3 years and a final
salvage value of ₱320,000. How much could the company afford to pay for another
type of truck for the same purpose, whose life is 4 years with a final salvage value
of ₱400,000, if money is worth 4%?

72
College of Engineering Education
2nd Floor, B&E Building
Matina Campus, Davao City
Telefax: (082) 296-1084
Phone No.: (082)300-5456/300-0647 Local 133

In a Nutshell

Activity 1: Based from learnings you have acquired from the metalanguage to the
activities you have done, state below the key points you have learned. I have indicated
my lesson learned.

1. Capitalized cost is an application of perpetuity.


Your Turn

2. _______________________________________________________________
_______________________________________________________________
_______________________________________________________________
_______________________________________________________________
_______________________________________________________________
3. _______________________________________________________________
_______________________________________________________________
_______________________________________________________________
_______________________________________________________________
_______________________________________________________________
4. _______________________________________________________________
_______________________________________________________________
_______________________________________________________________
_______________________________________________________________
_______________________________________________________________
5. _______________________________________________________________
_______________________________________________________________
_______________________________________________________________
_______________________________________________________________
_______________________________________________________________

Q & A List
List 1: After series of discussion and exercises, kindly list down your questions or issues
in conjunction with the topics. Write the answers after clarification under the column
provided. This portion helps you in the review of concepts and essential knowledge.

Do you have any questions for clarification?


Questions/Issues Answers
1.
2.
3.

73
College of Engineering Education
2nd Floor, B&E Building
Matina Campus, Davao City
Telefax: (082) 296-1084
Phone No.: (082)300-5456/300-0647 Local 133

4.
5.

Keywords Index
Index: The keywords presented here will help you remember the concepts and essential
knowledge. Here are the few subject indices.

A C M O
Annual Cost Capitalized Cost Maintenance Operations Cost
Cost

R S
Replacement Salvage Value
Cost

74
College of Engineering Education
2nd Floor, B&E Building
Matina Campus, Davao City
Telefax: (082) 296-1084
Phone No.: (082)300-5456/300-0647 Local 133

Big Picture
Week 8-9: Unit Learning Outcomes-Unit 4 (ULO-4): At the end of the unit, you are
expected to:
e. Solve for the price of a bond at a given interest rate; and
f. Compute for the break-even point of a manufactured quantity.

Big Picture in Focus:


ULO-4a. Solve for the price of a bond at a given interest rate

Metalanguage
The most essential terms below are defined for you to have a better
understanding of this section in the course.

1. Bond. It is a written contract to pay a certain redemption value on a specified


redemption date and to pay equal dividends periodically.
2. Dividend. It is a sum of money that is paid regularly by a company to its
shareholders out of its profits. This can be issued in different forms, such as cash
payment, stocks, or any other form.
3. Face Value. It can also be referred to as Par Value or simply Par. It is the value
printed on a coin, banknote, postage stamp, ticket, etc., especially when less than
the actual value or intrinsic value.
3.1 For bonds, it is the amount paid to the holder at maturity.

Essential Knowledge
In ULO4a, we will discuss how to compute for a bond price at a given interest
rate.

1. Classification of Bonds. Bonds are classified into two (2):


a. Registered Bond. The name of the owner of this bond is recorded on the record
books of the corporation and interest payments are sent to the owner periodically
without any action on his part.
b. Coupon Bond. This has coupon attached to the bond for each interest payment
that will come due during the life of the bond. The owner of the bond can collect
the interest due by surrendering the coupon to the offices of the corporation or at
specified banks.

2. Methods of Bond Retirement. Take note of the following principle and formula in
calculating for the price of a bond.
a. The corporation may issue another set of bonds equal to the amount of bonds
due for redemption.

75
College of Engineering Education
2nd Floor, B&E Building
Matina Campus, Davao City
Telefax: (082) 296-1084
Phone No.: (082)300-5456/300-0647 Local 133

b. The corporation may set up a sinking fund into which periodic deposits of equal
amount are made. The accumulated amount in the sinking fund is equal to the
amount needed to retire the bonds at the time they are due.

• A bond is said to be redeemable at par if the redemption value C is equal


to the face value F.

• A bond is said to be redeemable at a premium if C>F.

• A bond is said to be redeemable at a discount if C<F.

𝐶 𝐷[(1 + 𝑖 )𝑛 − 1]
𝑃= +
(1 + 𝑖)𝑛 (1 + 𝑖)𝑛 𝑖

Where:
F= face value or par value of a bond
C= redemption value on a specified redemption date
r= bond rate or dividend rate
D= periodic dividend
𝐷=𝐹𝑥𝑟

i= investor’s interest rate of return


P= price of the bond at a given interest i

Example 4a.1: A ₱100,000, 6% bond, pays dividends semiannually and will be


redeemed at 110% on July 1, 2020. Find its price if bought on July 1, 2017, to yield an
investor 4%, compounded semiannually.

Given: Solution:
𝐹 = ₱100,000 First, we need to solve for the value of D:
𝐶 = 110% 𝑜𝑓 𝐹 𝐷 = 𝐹 𝑥 𝑟 = ₱100,000(0.03) = ₱3,000
0.06
𝑟 = 2 = 0.03 Secondly, compute for C. Based on the given, C= 110% of F:
0.04
𝑖= = 0.02 𝐶 = (1.1)(₱100,000) = ₱110,000
2
Lastly, we can now plug in all the required elements in the formula to solve for P:
𝑛 = 3(2) = 6
₱110,000 ₱3,000[(1+0.02)6 −1]
Required: 𝑃 = (1+0.02)𝑛 + (1+0.02)6(0.02)
𝑃 =?
𝑃 = ₱𝟏𝟏𝟒, 𝟒𝟖𝟏. 𝟏𝟒

76
College of Engineering Education
2nd Floor, B&E Building
Matina Campus, Davao City
Telefax: (082) 296-1084
Phone No.: (082)300-5456/300-0647 Local 133

Example 4a.1: What is the maximum amount an investor should pay for a 25-year bond
with a ₱20,000 face value 8% coupon rate (interest rate only paid semiannually)? The
bond will be kept to maturity. The investor’s effective annual interest rate for economic
decision is 10%.
Solution:
Given: First, we need to solve for the value of D:
𝐹 = ₱20,000
𝐷 = 𝐹 𝑥 𝑟 = ₱20,000(0.04) = ₱800
𝐶=𝐹 Secondly, compute for i:
0.08
𝑟 = 2 = 0.04
0.1 = (1 + 𝑖 )2 − 1
𝑖 = 10% 𝑎𝑛𝑛𝑢𝑎𝑙𝑙𝑦 𝑖 = 0.0488
𝑛 = 25(2) = 50 Lastly, we can now plug in all the required elements in the formula to solve for P:
₱20,000 ₱3,000[(1+0.0488)6 −1]
Required: 𝑃 = (1+0.0488)50 + (1+0.0488)6 (0.0488)
𝑃 =?
𝑃 = ₱𝟏𝟔, 𝟕𝟐𝟕

Self-Help: You can also refer to the sources below to help and guide you
to further understand the lesson

Park, Chan S. (2011), Contemporary Engineering Economics (5th Edition), New Jersey:
Pearson Education, Chapters 8, 9, 11

Blank, L T., (2012), Engineering Economy (7th Edition), New York: McGraw Hill,
Chapter 9, 10, 11, 12, 13, 16, 17

Sta. Maria, H., Engineering Economy (3th Edition), National Book Store

Panneerselvam, R., (2012), Engineering Economics (Eastern Economy Edition), New


Delhi: PHI Learning Private Limited

Lindeburg, M., (2014), Civil Engineering Reference Manual for the PE Exam (14th
Edition), California: Professional Publications, Inc.

Let’s Check

Activity 1: At this point, it is assumed that have gained enough knowledge to answer
the practice problems below. Copy and answer the following word problems and show
a clear solution for each.

1. A man paid ₱110,000 for a ₱100,000 bond that pays ₱4000 per year. In 20 years,
the bond will be redeemed for ₱105,000. What net rate of interest will the man obtain
on his investment?
77
College of Engineering Education
2nd Floor, B&E Building
Matina Campus, Davao City
Telefax: (082) 296-1084
Phone No.: (082)300-5456/300-0647 Local 133

2. A company issued 50 bonds of ₱1,000 face value each, redeemable at par at the
end of 15 years to accumulate the funds required for redemption, the firm
established a sinking fund consisting of annual deposits, the interest rate of the fund
being 4%. What was the principal in the fund at the end of the 12 th year?

3. A man wants to make 14% nominal interest compounded semiannually on a bond


investment. How much should the man be willing to pay now for a 12% ₱10,000-
bond that will mature in 10 years and pays interest semiannually?

Let’s Analyze

Activity 1: Solve the following problems and state all the givens and its requirement.

1. Mr. Sy bought a bond having a face value of ₱1,000 for ₱970. The bond rate was
14% nominal and interest payments made to him semiannually for a total of 7 years.
At the end of the seventh year, he sold the bond to a friend at a price that resulted
a yield of 16% nominal on his investment. What was the selling price?

2. A ₱20,000, thirty-year bond that was issued 25 years ago with a bond interest rate
of 4% per year, payable quarterly is for sale for ₱15,000. The market interest rate is
12% per year, compounded quarterly. (a) What are the interest payments associated
with this bond? (b) What is the present worth of the bond?

3. A bond which was issued 12 years ago has a maturity date of 30 years from the time
it was issued. The bond has a face value of ₱5,000 and an interest rate of 6% per
year payable annually. The market interest rate is 16% per year.

In a Nutshell

Activity 1: Based from the definition of the most essential terms in the study of
curriculum and the learning exercises that you have done, please feel free to write your
arguments or lessons learned below. I have indicated my lesson learned.
6. There is a profit if the redemption value is greater than the face value.
Your Turn
7. _______________________________________________________________
_______________________________________________________________
_______________________________________________________________
_______________________________________________________________
_______________________________________________________________
8. _______________________________________________________________
_______________________________________________________________
_______________________________________________________________

78
College of Engineering Education
2nd Floor, B&E Building
Matina Campus, Davao City
Telefax: (082) 296-1084
Phone No.: (082)300-5456/300-0647 Local 133

_______________________________________________________________
_______________________________________________________________
9. _______________________________________________________________
_______________________________________________________________
_______________________________________________________________
_______________________________________________________________
_______________________________________________________________
10. _______________________________________________________________
_______________________________________________________________
_______________________________________________________________
_______________________________________________________________
_______________________________________________________________

Q & A List
List 1: After series of discussion and exercises, kindly list down your questions or issues
in conjunction with the topics. Write the answers after clarification under the column
provided. This portion helps you in the review of concepts and essential knowledge.

Do you have any questions for clarification?


Questions/Issues Answers
1.
2.
3.
4.
5.

Keywords Index
Index: The keywords presented here will help you remember the concepts and essential
knowledge. Here are the few subject indices.

B D F P
Bond Dividend Face value Par value

R
Redemption
value

Rate of return

79
College of Engineering Education
2nd Floor, B&E Building
Matina Campus, Davao City
Telefax: (082) 296-1084
Phone No.: (082)300-5456/300-0647 Local 133

Big Picture in Focus:


ULO-4b. Compute for the breakeven point of a manufactured quantity.

Metalanguage
This section will serve as your word bank where the most essential terms relevant
to the breakeven analysis will be operationally defined to establish a common frame of
reference. Please refer to these definitions in case you will find it difficult to understand.
1. Breakeven Point. It is the value of the variable for which the costs for the
alternatives will be equal.
1.1 It is the point at which total cost and total revenue are equal, that is “even”. It
means that there is no net loss or gain, and an individual has “broken even”.
1.2 The breakeven point is the level of production at which the costs of production
is equal to the revenues for a product.
2. Breakeven Analysis. It is a method of determining when costs exactly equal
revenue. If the manufactured quantity is less than the breakeven quantity, a loss is
experienced. If the manufactured quantity is greater than the breakeven quantity, a
profit is incurred. It also referred to as cost analysis.

Essential Knowledge
In ULO-4b, you are expected to compare two or more alternatives that are
affected by a common variable and show which one will be more economical in the long
run. All concepts discussed in this course will be applied to weigh the given alternatives.
1. Breakeven Point. It is the quantity of production at which income is equal to the
total cost. Study the given formulas and principles below to understand further the
definition of breakeven concept.
𝑹 = 𝒑𝑵
𝑪 = 𝒇 + 𝒂𝑵

If there are no changes in inventory, the breakeven point can be calculated by the
combination of 𝑹 and 𝑪.
𝒇
𝒇 + 𝒂𝑵 = 𝒑𝑵 𝑵=
𝒑−𝒂
where:
𝑓 = fixed cost which does not vary with production
𝑎 = an incremental cost which is the cost to produce one additional item. It can also be called
“marginal cost” or “differential cost”
𝑁 = breakeven point or quantity produced and sold for breakeven
𝑝 = incremental revenue or selling price per unit
𝑅 = total revenue
𝐶 = total cost

80
College of Engineering Education
2nd Floor, B&E Building
Matina Campus, Davao City
Telefax: (082) 296-1084
Phone No.: (082)300-5456/300-0647 Local 133

2. Breakeven Chart. It is a graphical representation of breakeven analysis. This plots


the sales revenue, different costs, and helps identify the breakeven point and margin
of safety.

Example 4b.1: The cost of producing a computer diskette is as follows: Material cost is
₱7.00 each, labor cost is ₱2.00 each, and other expense is ₱1.50 each. If the fixed cost
is ₱69,000 per month, how many diskettes must be produced each month for breakeven
if each diskette is worth ₱45.00?

Given: Solution:
𝑓 = ₱69,000 per month Use:
𝑎 = ₱7.00 + ₱2.00 + ₱1.50 𝑓
𝑁= 𝑝−𝑎
= ₱10.50
𝑝 = ₱45.00 𝑁=
₱69000
= 2000
₱45−₱10.50
Required: 𝑵 =? Therefore, 2000 diskettes must be produced each month for breakeven.

Example 4b.2: A company that manufactures electric motors has a production capacity
of 200 motors a month. The variable costs are ₱150.00 per motor. The average selling
price of the motors is ₱275.00. Fixed costs of the company amount to ₱20,000 per
month, which includes taxes. How many motors must be sold each month to
breakeven?

Given: Solution:
𝑓 = ₱20,000 per month Use:
𝑎 = ₱150.00 𝑓
𝑁= 𝑝−𝑎
𝑝 = ₱275.00
₱20000
𝑁= = 160
Required: 𝑵 =? ₱275−₱150

Therefore, 160 motors must be produced each month for breakeven.

81
College of Engineering Education
2nd Floor, B&E Building
Matina Campus, Davao City
Telefax: (082) 296-1084
Phone No.: (082)300-5456/300-0647 Local 133

NOTE: More examples will be posted on the Blackboard LMS Forum.

Self-Help: You can also refer to the sources below to help and guide you
to further understand the lesson

Park, Chan S. (2011), Contemporary Engineering Economics (5th Edition), New Jersey:
Pearson Education, Chapters 8, 9, 11
Blank, L T., (2012), Engineering Economy (7th Edition), New York: McGraw Hill,
Chapter 9, 10, 11, 12, 13, 16, 17
Sta. Maria, H., Engineering Economy (3th Edition), National Book Store
Panneerselvam, R., (2012), Engineering Economics (Eastern Economy Edition), New
Delhi: PHI Learning Private Limited

Lindeburg, M., (2014), Civil Engineering Reference Manual for the PE Exam (14th
Edition), California: Professional Publications, Inc.

Let’s Check

Activity 1: Copy and answer the following word problems and show a clear solution for
each.

4. ABC Corporation manufactures bookcases that sells for ₱65,000 each. It costs ABC
₱35,000 per year to operate its plant. This sum includes rent, depreciation charges
on equipment, and salary payments. If the cost to produce one bookcase is ₱50, how
many cases must be sold each year for ABC to avoid taking a loss?

5. Determine the breakeven point in terms of numbers of units produced per month
using the following data:

Selling Price per unit……………………………………… ₱ 600


Total monthly overhead expenses…………………….... ₱ 428,000
Labor cost………………………………………………….. ₱ 115
Cost of materials………………………………………….. ₱ 76
Other variable cost………………………………………... ₱ 2.32

82
College of Engineering Education
2nd Floor, B&E Building
Matina Campus, Davao City
Telefax: (082) 296-1084
Phone No.: (082)300-5456/300-0647 Local 133

Let’s Analyze

Activity 1: Solve the following problems and state all the givens and its requirement.

1. A chemical company is trying to decide which type of coating to use inside its chemical
storage tanks. A bituminous coating will cost $6,000 initially and will last for 10 years if
'touched up' at the end of year 4. An epoxy coating will cost $15,000, but it will last for 10
years with no other maintenance required. At an interest rate of 15% per year, how much
would the touch-up cost have to be in order for the alternatives to break even?

2. The cost of a machine for producing a certain part is $40,000. The machine is expected to
have a maintenance cost of $14,000 and an $8,000 salvage value after its 5-year economic
life. If the variable cost for producing the part is $1.50 per unit and the part can be sold for
$4.00 per unit, how many parts per year must the company sell in order to breakeven at an
interest rate of 12% per year?

In a Nutshell

Activity 1: Based from learnings you have acquired from the metalanguage to the
activities you have done, state below the key points you have learned. I have indicated
my lesson learned.

6. Breakeven analysis is concerned with finding the point at which revenues and
costs are exactly equal.
Your Tur

7. _______________________________________________________________
_______________________________________________________________
_______________________________________________________________
_______________________________________________________________
_______________________________________________________________
8. _______________________________________________________________
_______________________________________________________________
_______________________________________________________________
_______________________________________________________________
_______________________________________________________________
9. _______________________________________________________________
_______________________________________________________________
_______________________________________________________________
_______________________________________________________________
_______________________________________________________________
10. _______________________________________________________________
_______________________________________________________________
_______________________________________________________________

83
College of Engineering Education
2nd Floor, B&E Building
Matina Campus, Davao City
Telefax: (082) 296-1084
Phone No.: (082)300-5456/300-0647 Local 133

_______________________________________________________________
_______________________________________________________________

Q & A List
List 1: After series of discussion and exercises, kindly list down your questions or issues
in conjunction with the topics. Write the answers after clarification under the column
provided. This portion helps you in the review of concepts and essential knowledge.

Do you have any questions for clarification?


Questions/Issues Answers
1.
2.
3.
4.
5.

Keywords Index
Index: The keywords presented here will help you remember the concepts and essential
knowledge. Here are the few subject indices.

B F L P
Breakeven Point Fixed Cost Loss Profit

R V
Revenue Variable Cost

84

You might also like